PDA

View Full Version : The time travel challange - Can it be done without plot holes?



taltamir
2009-11-24, 06:27 AM
I abhor time travel plots. There are several "internally consistent" time travel types... when you get a time travel story it typically uses an inherently inconsistent model or it switches models midway.
Even if it manages to avoid the most obvious pot holes, it still ends up full of plot holes...

So here is the challenge. Can someone name any single story, comic, book, movie, or whatever; which contains time travel and is not left with gaping plot holes caused by said time travel in the first place?

When you post the name of such a story, expect it to be scrutinized by people hell bent on disproving your claim.

Also, feel free to suggest "solutions" to said plot holes that will not wreck the original plot.

raitalin
2009-11-24, 06:46 AM
Primer (http://en.wikipedia.org/wiki/Primer_(film)) might be the best example. It also leads to a extraordinarily convoluted plot.

kamikasei
2009-11-24, 06:59 AM
Sam Hughes tries to explore consistent time travel in a few of his stories. He's got an article (http://qntm.org/?models) discussing different models as used in fiction. (However, that article reveals which of his stories use which models, so I would recommend you read it after you read my story recommendations in the spoiler below, unless you're sure you're totally uninterested in reading them - and they're pretty interesting.)

He also has a pair (http://qntm.org/?coffin) of articles (http://qntm.org/?primer) on Primer, as mentioned by an earlier poster.

I'll only say that these stories involve time travel, which is no huge spoiler. I would recommend reading them before the article on different models in fiction, as that way you get the relevant explanation as part of the story.

There are a bunch of stories linked by the character Ed (http://qntm.org/?ed). They're worth reading in order. The first one with time travel is Be Here Now (http://qntm.org/?beherenow), which is more-or-less self-contained. Then once time travel is established in the setting it's relevant to the three remaining stories (which I'd regard as one long, extended story). (The structure of the whole Ed saga is essentially a handful of one-offs that establish some amusing things, then some large stories that are all related to one another and include references back to the earlier, establishing stories, so it pays to just start at the beginning and read the lot.)

The Four-and-a-Halfth Planet (http://qntm.org/?destroyed) is a non-Ed story about time travel, using a different model.

Causal Noose (http://qntm.org/?causal) is kind of about time travel too, ish.

Blayze
2009-11-24, 08:15 AM
Also, feel free to suggest "solutions" to said plot holes that will not wreck the original plot.

Despite how complicated people like to make the whole thing, it really boils down to one of two rules:

1) The past cannot be changed.
2) The past can be changed.

The "alternate timeline" theory annoys me, as it's just a cheap way of explaining paradoxes. As far as I'm concerned, the Grandfather Paradox has two answers, one for each of the above rules.

1) Because the past cannot be changed, nothing happens in the past that didn't already happen the first time around -- which means if you went back in time in order to slaughter your family tree, you'd fail and possibly become a family anecdote about the time Great Uncle Albert murdered someone screaming about science.

2) The act of travelling through time puts the person who did take the journey "outside" of causality to some extent in their current form -- meaning that even if you went back in time and killed your past self, you as you are now would still exist because you're a temporal anomaly anyway. Unfortunately, as a result your entire life between those two points would vanish, leaving the current you as the only person who remembers you. It also allows protection against your actions preventing you from taking the trip through time due to other reasons, as well.

Examples:

The first I used in earlier versions of Estra, where everything Luja did was predestined -- the knowledge of which, as well as his actions in an attempt to improve the life of his past self, eventually turned him into the Big Bad he became, who then went back through time...

The second model is what I use currently. The Luja who is the Big Bad has altered the life of his past self countless times in an attempt to improve his own life. Paranoid beyond belief, he took an insane number of precautions to ensure that, despite all the changes he made each time, his past self would eventually become his current self -- which leads to an awesome finale in my eyes, where Protagonist!Luja goes back in time and kills his child self in a desperate attempt to get rid of Antagonist!Luja, but fails as both he and his future self are 'outside' the timeline, so to speak.

Serpentine
2009-11-24, 08:15 AM
I read a website ages ago that analysed time travel in film. I tracked it down 'specially for you. Appreciate it. APPRECIATE IT!
Linky-poos. (http://www.mjyoung.net/time/index.htm)

I don't know whether he thought any of them were particularly well done. 12 Monkeys, perhaps.

Jamin
2009-11-24, 08:17 AM
No it can't Time Travel will never ever work as tool to strengthen story it will always weaken it. Stuff about time travel can be good but they are good in spite of time travel never because of it.

kamikasei
2009-11-24, 08:22 AM
No it can't Time Travel will never ever work as tool to strengthen story it will always weaken it. Stuff about time travel can be good but they are good in spite of time travel never because of it.

"Worsens the plot" is not "is/has a plot hole".


The "alternate timeline" theory annoys me, as it's just a cheap way of explaining paradoxes. As far as I'm concerned, the Grandfather Paradox has two answers, one for each of the above rules.

Er, is there any reason besides personal taste that you dismiss the "alternate timeline" approach? It certainly seems a valid third possibility, or rather alternate version of your second rule ("the past" can be changed, but it's not your past).

Serpentine
2009-11-24, 08:26 AM
As an aside, that website I posted has the following diagram to demonstrate different theories of time travel.

http://www.mjyoung.net/imgsrc/anom.jpg

You'll have to go to the site to see what it means, though. It's pretty complicated.

Cyrion
2009-11-24, 10:51 AM
Tau Zero by Poul Anderson and The Forever War by Joe Haldeman both involve relativistic time travel and may avoid some of the plot holes you complain about, though since they don't involve slipping into the past to muck about, they may not fit the spirit of the definition for this thread. Still both good stories, though.

warty goblin
2009-11-24, 11:05 AM
The real problem that I've always had with time travel is that of mass. Consider, you go back in time .1 seconds, then wait .1 seconds. There are now two of you. You plus yourself go back in time .1 seconds, then wait .1 seconds. There are now four of you. You plus yourselfx3 go back in time .1 seconds, then wait .1 seconds. There are now eight of you.*

You have, in short, an exponential mass generating device with a more or less arbitrarily short generation time. Now generating buckets upon buckets of time clones is useful, but it gets real fun when you start to play with things like fuel. Consider the above example, only everybody is carrying a gallon of gasoline. By the end of .1 seconds from anybody else's perspective, you have eight gallons.


Screw going back in time to kill Hitler. The real potency of time travel is in manufacturing.


*This makes the assumption that time travel has some way of solving colision issues. so when you go back in time .1 seconds, then age back up to the present, you don't appear within yourself. If it doesn't, then you can use repeated iterations of this trick to create objects of arbitrary density out of dirt.

Zanaril
2009-11-24, 11:22 AM
The real problem that I've always had with time travel is that of mass. Consider, you go back in time .1 seconds, then wait .1 seconds. There are now two of you. You plus yourself go back in time .1 seconds, then wait .1 seconds. There are now four of you. You plus yourselfx3 go back in time .1 seconds, then wait .1 seconds. There are now eight of you.*

But wouldn't that point-one-second-in-the-future you then go back in time, leaving only one of you? Any overlap would only be temporary, because assuming you can't change the future, the future you will allways eventually go back in time.

Cespenar
2009-11-24, 11:29 AM
I think the safest and most sensible (if ever) way of time travel is what I call the "conservation of self". If you travel to the past, for example, you don't see your past self, because you traveled within your own body. There are no alternate timelines, and you can affect the world as much as you do in normal life, because you haven't outwitted causality (which is stupid, actually) and still exist in it.

This also limits the travel to the extent of your lifetime, which is a side bonus (keeping things small).

Totally Guy
2009-11-24, 11:31 AM
Australian webcomic Magellan (its about a superhero school) handled a time travel plot very well indeed. That particular arc is very long.

Their rules were that time travelling split the timestream so that both realities continued. Anything the time travellers did had no effect on their own universe but the changed past had full effect on the "past" universe.

It pretty much told the story of what would have happened without the time travel through the exposition of the time travellers. Then it is made clear that the future of the time travellers is not be the story you're reading.

truemane
2009-11-24, 11:33 AM
12 Monkeys is my favourite 'the past cannot be changed' narrative. By going back to try and change the past, Cole actually caused it to accur and even generated one of his own childhood memories. Good times.

And once you admit the past CAN be changed, the major issue is that once it IS changed it's "always" been that way and no one could possibly remember the way it "used to be."

EXAMPLE: I go back in time and tell my Dad to invest heavily in Apple Stocks (or short-sell Enron, or whatever). I bring along some manner of proof of who I am so that he'll believe me. He does so. He generates enormous wealth.

Now I'm a rich kid. And assuming we hand-wave the paradox of me never having gone back in time to change the past "now" that I'm rich, I still have no memory of the way I "used to be."

Heh. Temporal references are fun when discussing time travel.

My wife thinks that alien visitations are actually our future selves coming back to study our past selves, much as we would with/to the Neanderthals if we had a time machine. But then, my wife married me, thereby casting her judgement into serious question.

EDIT: Also, Primer is indeed an excellent film. Convoluted as all hell, but VERY entertaining.

Joran
2009-11-24, 11:46 AM
Here's an article about time travel from a physicist who oddly enough finds the time travel in The Time Traveler's Wife adheres pretty well to what physicists think time travel would look like.

http://www.slate.com/id/2225223

1. No parallel universes, this is the only universe we have.
2. Can't go back past the creation date of the time travel machine.
3. No grandfather paradox, which leads to an odd thing about free will...

Basically, Kip Thorne, the theoretical physicist who came up with wormholes, came up with a seemingly solution to the grandfather paradox.

There's a thought experiment very similar to the grandfather paradox called the impossible cue shot. Imagine you have a time machine aperture at the end of a pool table and place the time machine exit at another place on the table. Set it up so that when you fire the cueball at the entrance, it'll exit at just the right time and speed to intercept the cueball and knock it away from the entrance so that it nevers enters the entrance. What happens when you fire the cueball at the entrance?

The compromise is that as the cueball goes towards the entrance, another cueball comes out of the exit at a different angle than you intended and redirects the cueball at a different angle into the entrance, which exits at the angle of the observed cueball.

How this would work with humans and free will, that's for the writers to decide up until we can do it.

Joran
2009-11-24, 12:19 PM
The real problem that I've always had with time travel is that of mass. Consider, you go back in time .1 seconds, then wait .1 seconds. There are now two of you. You plus yourself go back in time .1 seconds, then wait .1 seconds. There are now four of you. You plus yourselfx3 go back in time .1 seconds, then wait .1 seconds. There are now eight of you.*

You have, in short, an exponential mass generating device with a more or less arbitrarily short generation time. Now generating buckets upon buckets of time clones is useful, but it gets real fun when you start to play with things like fuel. Consider the above example, only everybody is carrying a gallon of gasoline. By the end of .1 seconds from anybody else's perspective, you have eight gallons.


Screw going back in time to kill Hitler. The real potency of time travel is in manufacturing.


*This makes the assumption that time travel has some way of solving colision issues. so when you go back in time .1 seconds, then age back up to the present, you don't appear within yourself. If it doesn't, then you can use repeated iterations of this trick to create objects of arbitrary density out of dirt.

After all your iterations, you'll have those 8 in the past at that particular time but in the future you'll eventually get back to only one of you (the one who decided not to step into the time machine again). If you use the gas can, then the iterations of you after you use that gas can won't have the gas can.

You can also solve it by having the time travel machine use up energy equal to the mass of you, so that the conservation of mass and energy is the same. I never was too good at physics =P

Blayze
2009-11-24, 12:39 PM
Er, is there any reason besides personal taste that you dismiss the "alternate timeline" approach?

Just personal taste, really. On a related note, I would *love* to see a scene where the heroes travel to an alternate reality -- only to discover all too late that the Earth wasn't in the exact same place. Whoops.

warty goblin
2009-11-24, 12:49 PM
After all your iterations, you'll have those 8 in the past at that particular time but in the future you'll eventually get back to only one of you (the one who decided not to step into the time machine again). If you use the gas can, then the iterations of you after you use that gas can won't have the gas can.

Right, any iterations after the final one uses the gas can won't be able to, but so what? I'm still able to generate energy in the past. This could then be used to charge a battery, or compress a gas or some other energy storage method. Then back in the present, I'd end up with a charged battery for the cost of running the time machine, plus one can of gas. Clearly to make this profitable, you want more than eight generations, but you get the idea.

Note that this assumes the past can be changed. To me, this is obvious from the construction, because I have mass in the past, and therefore the past is altered to begin with. The problem with most 'can't change the past' constructions is that it still allows the time traveler mass (specifically mass from the future), and thus by traveling backwards they do in fact alter the past by their mere presense. It might not be enough to have any major effects, but the potential is there. I'm simply exploiting that.

Actually my original construction was flawed. Here's how it should work:
1) I travel ten minutes into the past, assuming that ten minutes previous to my time travel, the time machine already existed.
2) In the past I wait some increment of time t less than ten minutes. Then I go through the time machine, setting it to take me t minutes into the past again.
3) There are now two copies of me ten minutes ago (well, three if you count the original, but he's off having fun, not screwing with time machines). Only one of these has mass from the present.
4) Again me and myself wait t minutes, then travel t minutes back into the past.
5) Now there are four copies of me ten minutes ago. But three of them have all of their mass taken from 10 - t minute ago.
6) Repeat this process until I have an arbitrary number of me's. Virtually all the mass is coming from sometime between ten minutes ago and the present. I can now utilize all of this mass in whatever way I see fit.


You can also solve it by having the time travel machine use up energy equal to the mass of you, so that the conservation of mass and energy is the same. I never was too good at physics =P

The problem still isn't removed though, since by traveling into the past, you took mass from the present and moved it to the past. There is now more mass in the past, and conservation has been violated. And energy isn't really used in the sense of ceasing to exist, using energy simply increases entropy. But it's all still there.

kamikasei
2009-11-24, 12:52 PM
I think the safest and most sensible (if ever) way of time travel is what I call the "conservation of self". If you travel to the past, for example, you don't see your past self, because you traveled within your own body. There are no alternate timelines, and you can affect the world as much as you do in normal life, because you haven't outwitted causality (which is stupid, actually) and still exist in it.

What, so you're sending your mind back in time to overwrite the one that inhabited your past body? Leaving aside the dualism, that doesn't really solve anything. The paradoxes of time travel are not "if you shake hands with your past self, the universe will explode!". What you describe does nothing to prevent, for example, destroying the time machine that you would have used to travel back in time, thus preventing your journey from ever occurring.

Could you explain what solution you think the model you describe does provide? Because I'm not seeing it.

Nerd-o-rama
2009-11-24, 01:23 PM
Well, warty, Conservation of Mass (and Energy) essentially states that the amount of Mass+Energy (including entropy) in the Universe is the same at all times. The Second Law of Thermodynamics also states that as time increases, entropy increases (and therefore useable energy decreases proportionally).

What a time machine would need to do in order to not violate these things is somehow replace the matter and energy taken out of the universe in the "present" with matter and energy that it takes out of the "past" (as well as immediately converting the energy component into entropy so as not do decrease the relative amount of entropy in the "present" Universe). This sets up all kinds of other problems, of course, and it does allow you to manipulate where in the Universe matter and energy are in the past, but it does work around the thermodynamic violations.

Assuming, of course, that the Laws of Thermodynamics are in fact true. If time travel's possible, of course, then all bets are off.

And I would like to point out that the only real problem with time travel is going to the past. Getting to the future quicker than normal is just a matter of moving at relativistic speeds.

chiasaur11
2009-11-24, 01:31 PM
Well, warty, Conservation of Mass (and Energy) essentially states that the amount of Mass+Energy (including entropy) in the Universe is the same at all times. The Second Law of Thermodynamics also states that as time increases, entropy increases (and therefore useable energy decreases proportionally).

What a time machine would need to do in order to not violate these things is somehow replace the matter and energy taken out of the universe in the "present" with matter and energy that it takes out of the "past" (as well as immediately converting the energy component into entropy so as not do decrease the relative amount of entropy in the "present" Universe). This sets up all kinds of other problems, of course, and it does allow you to manipulate where in the Universe matter and energy are in the past, but it does work around the thermodynamic violations.

Assuming, of course, that the Laws of Thermodynamics are in fact true. If time travel's possible, of course, then all bets are off.


That's what they did in Transformers, anyway.

Nerd-o-rama
2009-11-24, 01:32 PM
That's what they did in Transformers, anyway.Really? Neat. Only watched a little bit of Beast Wars when I was 10 or so, didn't get to the part where they explained the time travel, or watch any other shows.

chiasaur11
2009-11-24, 01:35 PM
Really? Neat. Only watched a little bit of Beast Wars when I was 10 or so, didn't get to the part where they explained the time travel, or watch any other shows.

Not in the shows, sadly.

Just in the UK comic. And the conservation of matter was only kept up for a while.

Admittedly, when it broke down, it caused demons to come pouring out of rifts in reality, mass destruction, and frequent minor character deaths, so the hard science is... less.

Still, Furman tried.

zyphyr
2009-11-24, 02:37 PM
Babylon 5 managed internally consistent time travel.

Cespenar
2009-11-24, 02:38 PM
What, so you're sending your mind back in time to overwrite the one that inhabited your past body? Leaving aside the dualism, that doesn't really solve anything. The paradoxes of time travel are not "if you shake hands with your past self, the universe will explode!". What you describe does nothing to prevent, for example, destroying the time machine that you would have used to travel back in time, thus preventing your journey from ever occurring.

Could you explain what solution you think the model you describe does provide? Because I'm not seeing it.

I didn't quite get the "problem" you're referring that my model should have solved, but I'll try to explain as much as I understood.

First off, there's only one timeline, so that makes things pretty simple to start with. Second, the timeline continues to be one-dimensional, so there is no way for any paradoxes to happen. For example, if I turn back and, let's say, prevent the time machine from being built, then the time machine simply isn't built, it doesn't make you disappear because you erased your past or something like that.

To simplify it further, let's say that it's like the save/load system. You can only load an instance that you have experienced before. Apart from your knowledge, everything gets reset/overwritten/rewound/what-you-say. That's that, more or less. No paradox.

kamikasei
2009-11-24, 02:49 PM
Your knowledge is a paradox.

This isn't time travel, it's solipsism or a Matrix.

If something that is you is somehow traveling back in time and acting in the past with knowledge of the future, with the power to change the past and thus prevent that future, that creates a paradox.

golentan
2009-11-24, 02:49 PM
John Varley's "Mammoth" was internally consistent.

Everything hung together properly, there were no gaping plot holes, and there were no paradoxes/alternate timelines.

This was resolved by granting the characters no knowledge of how time travel works (they find a broken time machine at the beginning, they try to duplicate it, fail, and then one of the models starts working when whacked with a sledgehammer for no explicable reason and goes on to become the template they used at the end of the book), and next to no free will (sure, they can make decisions. But known facts have a way of asserting themselves. Unknown facts vary, though it's unclear if this is free will in the absence of discovery of predetermination (knowing it happened makes it so?) or if they are in fact conforming to predetermination but simply are unaware of it, a philosophical conundrum Varley has explored before). The time machine isn't really controllable, simply hurling characters around as it feels like to conform to known events (the sole exception isn't duplicable, and is implied to possibly be divine intervention).

It's not the greatest book, but it meets the Criteria.

Asheram
2009-11-24, 03:00 PM
The real problem that I've always had with time travel is that of mass. Consider, you go back in time .1 seconds, then wait .1 seconds. There are now two of you. You plus yourself go back in time .1 seconds, then wait .1 seconds. There are now four of you. You plus yourselfx3 go back in time .1 seconds, then wait .1 seconds. There are now eight of you.*

You have, in short, an exponential mass generating device with a more or less arbitrarily short generation time. Now generating buckets upon buckets of time clones is useful, but it gets real fun when you start to play with things like fuel. Consider the above example, only everybody is carrying a gallon of gasoline. By the end of .1 seconds from anybody else's perspective, you have eight gallons.


Screw going back in time to kill Hitler. The real potency of time travel is in manufacturing.

Not really, becuase it's not 8 different cans of gas, it's one can of gas in eight different timestates and locations.

The problem with the temporal paradox is that once an item have been sent back into time, it occupies two places at once, and what affects the "younger" (eg. the item before it's sent back in time) item "should" affect the other.
This is why dimensional timetravel is much more believable.

A paradox by the first setting, pure timetravel, would be; If you have X, and send X back in time, then we have X1 and X2, Primary (older) and secondary (younger).
If X1 affects X2 in the way X2 it's Not sent back in time, then we have a paradox, since how could then X1 exist?

if we use Dimensional timetravel on the other hand, then we have X1 from dimension Y1.
And X2 From dimension Y2.
You're just sending X1, to dimension Y2, without affecting dimension Y1 in any way other than that X1 no longer exists in that dimension.
In dimensional timetravel, there can be no paradoxes.

Douglas
2009-11-24, 03:00 PM
Actually my original construction was flawed. Here's how it should work:
1) I travel ten minutes into the past, assuming that ten minutes previous to my time travel, the time machine already existed.
2) In the past I wait some increment of time t less than ten minutes. Then I go through the time machine, setting it to take me t minutes into the past again.
3) There are now two copies of me ten minutes ago (well, three if you count the original, but he's off having fun, not screwing with time machines). Only one of these has mass from the present.
4) Again me and myself wait t minutes, then travel t minutes back into the past.
5) Now there are four copies of me ten minutes ago. But three of them have all of their mass taken from 10 - t minute ago.
6) Repeat this process until I have an arbitrary number of me's. Virtually all the mass is coming from sometime between ten minutes ago and the present. I can now utilize all of this mass in whatever way I see fit.
Here's how that would work under a "can't change the past" model:
1) Ten minutes before your planned trip to the past, an enormous horde of copies of yourself shows up.
2) Over the next ten minutes, the horde thins out as its members travel to the past.
3) All but one member of the horde is gone and you travel to the past.
4) You find yourself as part of the horde that appeared in step 1.
5) You cycle through being each and every member of the horde one at a time until you finally end up being the one that didn't travel to the past.
6) The single you looks back on the bizarre 10 minute interval that you lived through a large number of times.

Any possible use you could put that mass towards will have to be temporary, because it will all go away again when your time copies depart. It is not possible for them to not depart because that would prevent their arrivals.

Cespenar
2009-11-24, 03:45 PM
Your knowledge is a paradox.

This isn't time travel, it's solipsism or a Matrix.

If something that is you is somehow traveling back in time and acting in the past with knowledge of the future, with the power to change the past and thus prevent that future, that creates a paradox.

Not necessarily. Knowledge is only an assortment of data. When you travel to the past, what knowledge you carry isn't from the "future", it's knowledge of a possibility. The moment you travel back, that "back" becomes the new "now", and that old "future" becomes nonexistent.

And yeah, though I can see how you linked it with solipsism, it isn't actually that. And your "this isn't time travel" phrase intrigues me. Does time travel actually exist that I seem to be digressing from it?

Anyway, despite all these, you probably realize that what I'm trying to explain is simply a theory, and (probably) no more possible than the other time travel theories out there. Still, it makes more sense to me than that old Back to the Future style "you unmake yourself!" jazz.

kamikasei
2009-11-24, 04:16 PM
Not necessarily. Knowledge is only an assortment of data. When you travel to the past, what knowledge you carry isn't from the "future", it's knowledge of a possibility. The moment you travel back, that "back" becomes the new "now", and that old "future" becomes nonexistent.

And yeah, though I can see how you linked it with solipsism, it isn't actually that. And your "this isn't time travel" phrase intrigues me. Does time travel actually exist that I seem to be digressing from it?

Either that data is coming from somewhere - from a future that you're then saying never actually existed, which means paradox - or it's not, in which case I don't know what you're trying to say at all. If the data isn't being somehow conveyed from one time to a past time, where's the time travel? If it is, how is there less paradox?

The reason I link it to solipsism is that, unless you want to say that your own mind is the only real thing and you can essentially decide that the world should revert to a state from some time in the past while you remain unchanged, then for your mind to be sent back in time is not different in any important way than for your body to be. It might get around some issues of conservation of this or that if you can't send back objects but only rearrange them in the past, so that you can effectively overwrite your past self's brain with your current mental makeup - but all the paradoxes of time travel still ensue.

Specifically, if you go back in time under this model and prevent the time machine you used from being built, why doesn't that create a paradox?


Anyway, despite all these, you probably realize that what I'm trying to explain is simply a theory, and (probably) no more possible than the other time travel theories out there. Still, it makes more sense to me than that old Back to the Future style "you unmake yourself!" jazz.

Back to the Future's model made no sense at all. It's not really a substantive basis for comparison.

Moff Chumley
2009-11-24, 04:31 PM
-All You Zombies- (http://en.wikipedia.org/wiki/—All_You_Zombies—)is internally consistent. A little creepy, but consistent. :smalleek:

Cespenar
2009-11-24, 04:54 PM
It doesn't create a paradox because you overwrite the timeline when you "travel" in it, it doesn't overwrite you.

See, in a "normal" time travel, it would go like this:

1- Time machine is built.
2- You use it, traveling to the past.
3- You prevent 1 from happening.
4- Since 1 didn't happen, 2 and 3 cannot happen. Paradox.

In my theory:

1- Time machine is built.
2- You use it, traveling to the "past".
3- If you prevent the time machine from being built, you don't actually prevent "your" time machine. You prevent a "new" time machine. Even though the dates of 1 and 3 may be the same, 1 comes before 3 according to the real timeline.

So, let me make up a real calendar, to further hypothesize and clear things up.

Real 2009: Normal 2009: Time machine is built.
Real 2009: Normal 2009: Using it, you return to, let's say, 1999.
Real 2009: Normal 1999: You sabotage the blueprints of the time machine.
Real 2019: Normal 2009: You wait 10 years. Nothing really happens.

Jahkaivah
2009-11-24, 05:25 PM
No it can't Time Travel will never ever work as tool to strengthen story it will always weaken it. Stuff about time travel can be good but they are good in spite of time travel never because of it.

So Back to the Future should have been about a boy and his girlfirend going to the Prom and Bill and Ted should have been about two boys studying really hard for a history exam? :smalltongue:

JonestheSpy
2009-11-24, 05:38 PM
Anyone interested in a serious exploration of time travel must read Kage Baker's "The Company" novels.

The books are based on a pretty simple, logical premise that's pretty common in TT stories - you can't change the past because it's already happened, but there is a huge amount of room for action that isn't written down and doesn't contradict recorded history. Baker illustrates the ramifications of that more than anyone else I've heard of, though.

A corporation invents and patents time travel, and starts colonizing all of the past with its agents. They save works of art thought to be destroyed, get samples of plants and animals that are recorded as going extinct, study languages and cultures that are later wiped out, and preserve it all for the post-TT discovery future. Of course, it becomes clear before too long that the Company has goals and plans that are far less benevolent than that.

Great, great stuff, highly recommended. Check out her books here (http://www.kagebaker.com/).

kamikasei
2009-11-24, 06:08 PM
It doesn't create a paradox because you overwrite the timeline when you "travel" in it, it doesn't overwrite you.

Er. No. That makes no sense whatsoever. Sorry.

Let me put it this way. When did the events you remember happening, happen?

charl
2009-11-24, 06:25 PM
PS238, which is a comic (and now also exists as a webcomic somewhere) has a surprisingly consistent time travel plot for being a silly work about superhero 8 year olds. It uses a lot of time loops to make it work, but it does it really well.

Also Planetary has a short story arc about time travel that tries to portray it as realistic as possible. I'm not sure how solid the theories used are though, but they seem to line up with what some theoretical physicists have posited so...

Cespenar
2009-11-24, 06:27 PM
They happened in the "real" past. Which doesn't exist in the "normal" timeline. Real timeline > Normal timeline, to clarify. Real timeline goes forward no matter what, while normal timeline is changeable. You may have another look at the last part of my previous post.

I'm sorry too, these theories verge between the metaphysical and the fictional, and my inability to describe them really doesn't help.

JonestheSpy
2009-11-24, 06:34 PM
Let me put it this way. When did the events you remember happening, happen?

In the other timeline, of course.

Really, it's a perfectly valid solution to the paradox - there is a whole school of thought in quantum physics that posits that universes are constantly splitting off into different timestreams where the coin ended up landing on heads instead of tails.

kamikasei
2009-11-24, 06:41 PM
Then it sounds like you're just describing a rather needlessly obfuscated version of the "traveling back creates an alternate timeline splitting off from the point of your arrival" model.

Which is a perfectly valid model, but adding the complication that it's only your mind that's traveling back to hijack your old body adds nothing to its validity. To put it another way, the model would work just as well and contain exactly as many (zero) paradoxes if you traveled back in body as well as mind and could, in fact, shake hands with your past self.

Joran
2009-11-24, 06:46 PM
In the other timeline, of course.

Really, it's a perfectly valid solution to the paradox - there is a whole school of thought in quantum physics that posits that universes are constantly splitting off into different timestreams where the coin ended up landing on heads instead of tails.

Yes, the many worlds theory. It's one way to solve the headache inducing properties of quantum physics. It's a nice way to resolve temporal issues in stories, but, the physicist in the article I linked says relativity undermines the theory.

taltamir
2009-11-24, 07:06 PM
I read a website ages ago that analysed time travel in film. I tracked it down 'specially for you. Appreciate it. APPRECIATE IT!
Linky-poos. (http://www.mjyoung.net/time/index.htm)

I don't know whether he thought any of them were particularly well done. 12 Monkeys, perhaps.

I appreciate, thank you.

It will take me some time though to read through the entire thread worth of posts and that article (so many people post in these forums :P)

Soras Teva Gee
2009-11-24, 07:59 PM
I always loved this sort of problem: You take a ring that belonged to your grandfather and travel back in time, you give your grandfather the ring, you inherit the ring you gave your grandfather. Where did the ring come from?

Maybe we should just stop trying to reconcile time travel with out modern understanding of physics? Course if you simply abandon the idea that you can't have a circular chain of events.... but that's some serious mind screw on how we see things. I never see what mechanism is supposed to enforce 'reason' on reality though by not allowing X event to happen because it makes no sense. If X event happened and made no sense then its our notion of sense that is wrong.

On a lighter note I've always been pleasantly surprised at how well Futurama handled time travel when most of the series is Rule of Funny and if treated seriously would be worse then B-movies for abusing science.

golentan
2009-11-24, 08:16 PM
I always loved this sort of problem: You take a ring that belonged to your grandfather and travel back in time, you give your grandfather the ring, you inherit the ring you gave your grandfather. Where did the ring come from?

Maybe we should just stop trying to reconcile time travel with out modern understanding of physics? Course if you simply abandon the idea that you can't have a circular chain of events.... but that's some serious mind screw on how we see things. I never see what mechanism is supposed to enforce 'reason' on reality though by not allowing X event to happen because it makes no sense. If X event happened and made no sense then its our notion of sense that is wrong.

On a lighter note I've always been pleasantly surprised at how well Futurama handled time travel when most of the series is Rule of Funny and if treated seriously would be worse then B-movies for abusing science.

The really fun thing is that the ring has to be invulnerable, or renovated with an object outside the loop. Else it will be worn down over time. Which means you can take the ring and use it for some physics breaking machines if you can pry it loose from the time loop.

taltamir
2009-11-24, 08:35 PM
Maybe we should just stop trying to reconcile time travel with out modern understanding of physics?

we aren't. we are trying to make it consistent.
You want physics? time travel is impossible, period, end, no story!

The complaint is the plot holes, fallacies, and internal inconsistencies.

HamHam
2009-11-24, 09:11 PM
The time travel in Babylon 5 has no plot holes caused by the time travel (there are some inconsistencies with the original Babylon 4 episode but that's because the story changed in the like 2 seasons in between).

Really, any stable time loop should have no plot holes.

Mewtarthio
2009-11-24, 09:32 PM
I thought the time travel in Six Days a Sacrifice was pulled off pretty well. Of course, that's because it's in the Cosmic Horror genre, so it doesn't bother trying to explain it. Really adds to the whole "utterly unknowable" feel when simple things like "cause comes before effect" get reversed.

Serpentine
2009-11-24, 10:38 PM
I appreciate, thank you.Danke :smallsmile:

The aforementioned article solves a lot of time-travel issues by having multiple time travel episodes. The Harry Potter one demonstrates this very well and is particularly interesting (not least because the movie does it better than the book).

I might owe J. K. Rowling something of an apology. I had read the book and seen the movie, and was caught on the fact that Harry Potter cannot survive unless he survives. If we learned one thing from Bill and Ted's Bogus Journey, it is that only those who win get to come back in time to change the details. These are wonderful stories well told, but this is a temporal disaster--and I said as much. I had, however, missed a possibility, and if one essential detail can be accepted, the film just might work.

Original History
The point to remember in all of this is that Hermione, not Harry, is the primary time traveler. Harry is the tagalong. He goes with Hermione if he is there, but she is the one who must make the trip. She has the time turner.

With this point established, we may look at our first run through history: what happens if Hermione does not travel to the past? In order to be clear in each timeline, the Hermione for whom this is the first time through these few hours will be called the Original Hermione, and the one who has used the time turner to come from the future will be denoted as the Duplicate Hermione, because she is a temporal duplicate of herself.

The first point is that Buckbeak is executed. Our trio of troublemakers, Harry, Ron, and the Original Hermione don't actually see this, but they hear the axe fall and assume, correctly, that it is the neck of the hippogriff that is severed by it.

This creates the obvious problem, which any of my regular readers will spot immediately: Hermione is going to travel to the past specifically to save Buckbeak, and is going to succeed; but if she succeeds, surely she will know that Buckbeak was not executed, and so will not know that she needs to save him. However, the picture painted by the film suggests that the Original Hermione and companions who are leaving Hagrid's are unaware that Buckbeak has been saved by their later counterparts, and since McNair vents his frustration on one of Hagrid's beautifully big pumpkins, they hear essentially the same sound, or near enough that they do not realize the beast is still alive. Dumbledore, of course, knows that Buckbeak died, and will know that he was saved, but in classic Dumbledore wisdom, the wizard seems already to know what Hermione is going to do. Thus this will not be a problem.

What is more of a problem is that, moments before in the viewed history, the Duplicate Hermione hurls two stones or clods of dirt through an open window, breaking a pot and hitting Harry in the back of the head, to alert the Original trio to the fact that the executioner is coming. It is thought to be important to their situation that they not get caught at Hagrid's cottage. However, the warning does not seem to make any difference anyway, as they are don't really scramble to leave the cottage until after Cornelius Fudge knocks on the door. Thus we have a throwaway, an action that accomplishes nothing but give us a bit of a time game: Hermione thinks to throw the stone because she sees the stone in the garden and recognizes it as the one she picked up in the cottage. The stones mean nothing.

Once the group leaves the cottage, Sirius Black, as a dog, spots them in the yard and tackles Ron, while Professor Remus Lupin is looking at the map so that he sees Peter Pettigrew (in Ron's hands) and follows, and does so at a moment when Professor Severus Snape sees him and follows in turn. Those events play out much as in the film; the duplicates never venture outside the grounds, and so essentially await the return of the originals.

It is at that moment that the first serious difference occurs: Harry is torn to bits by the werewolf Remus Lupin. Sirius attempts to protect him, but is badly battered in the process. Hermione has nothing to use against a werewolf, and cannot save Harry at that moment. The best--perhaps the only--thing she can do is help Snape get Ron to the castle.

Sirius winds up down by the lake, where the Dementors find him and destroy him, sucking out his soul and leaving his body behind. It has been a bad day, with Buckbeak beheaded, Harry hashed, and Sirius suctioned.

This puts Hermione in Madam Pomfrey's treatment room, possibly being treated for her own minor injuries, at the right time for Dumbledore to recommend to her that she can save more than one life if she goes back three turns. She does so, alone, probably expecting to save Harry but not knowing quite how to do it. This concludes what we call the AB timeline, the original history, the way events must have played before anyone left from the future and arrived in the past.

Back to top of page.


Good Old Hermione
Before we consider the changes Hermione will have to make to create the second, CD, timeline, we should recognize that Hermione has been doing this all year, and that the book and the movie have both overlooked a minor point in this connection. Also, there is not a great deal of information available to tell us what has been happening. Hermione was not given the time turner to help Harry; she was given the time turner so that she could attend her incredibly overloaded course schedule, which included classes in different rooms at the same time. It is not clear how many such classes she had, or how long each class is; however, if we assume that she had to add two hours to each school day, five days per week, we have her working twenty-six hour days during the week and picking up an extra ten hours per week. Thus every five weeks she has gained two days on her peers. If school runs from early September to late May with a single break at Christmas for two weeks, we have almost thirty-five weeks, and Hermione has picked up almost fourteen days, two extra weeks of aging. This may be a conservative estimate. After all, with the extra classes she may want extra study time, and with the longer days she may need longer nights to catch up her sleep (how convenient to be able to sleep in and make the first class on time). It's not going to be a noticeable amount even if we stretch it to two months, but Hermione will be aging faster than everyone else.

This also means she is creating a series of anomalies--an original history in which she attends one class but not the other, and then an altered history in which she attends both classes. Any one of these in theory could go wrong; however, her intentions are quite clear, and should work. After all, it is not in this case that having missed a class she decides after the fact to fix that by traveling back in time to attend it instead (something she specifically determines not to do after Ron and Harry note her absence from a class), but rather that before attending either class she has decided to attend one and then travel back in time to attend the other as well. This being her intention, she should be able to maintain consistent N-jump outcomes through the school year, as long as nothing happens to derail the program completely.

Back to top of page.


Hermione Solos
With Dumbledore's words that she might save more than one life if she goes back three turns, Duplicate Hermione arrives in the early evening. Just as she does when Harry is with her, she works out that the first life to save is Buckbeak's, and she moves around behind Hagrid's house unnoticed. Her original self is still inside, and she wants to move her companions on their way, so she throws the rocks and manages to break the pot and hit Harry. This makes no difference, however, as they do not move until the approaching group knocks on the door anyway.

It is possible that Original Hermione might see Duplicate Hermione; but Original Hermione is aware that she can travel in time, and has enough on her mind at the moment that she'll probably not give it a second thought until later. That action replays in each subsequent history, as Duplicate Hermione steps out from cover to see her own hair, and Original Hermione gets a glimpse of herself ducking back under cover.

She has to save Buckbeak without Harry's help; but since she has no notion of having Harry's help anyway, she will do this. She will of course bow, as she was taught, and then probably use the dead ferrets in combination with the chain to lure the hippogriff into the forest out of sight.

Having saved Buckbeak, she now moves deeper into the forest and circles around to a spot from which she can more easily observe the Womping Willow, the spot from which Harry will emerge. Eventually he does so, and Remus, struck by the moonlight, transforms into the werewolf. This Duplicate Hermione has probably already considered her options, and so without hesitation she makes the wolf call--and then realizes that she has lured a werewolf directly toward her. She runs from this, hiding among the trees, and is saved by Buckbeak, who drives Lupin away from her.

However, Hermione is unaware that Harry, whom she has just saved from the werewolf, has run off after Sirius and is about to be killed by the dementors. By the time she knows this, it will be too late; Harry and Sirius will be dead. She will return to the castle somewhat dejected, but before she returns to the medical ward her Original self will depart, creating yet another altered history. The first altered history, the CD timeline, comes to a less than glorious end.

Back to top of page.


Take Two
The new Duplicate Hermione does not know that Harry was saved from Lupin by the wolf call from a previous duplicate Hermione; she only knows that she has to figure out how Harry got down to the lake, and somehow save his life. She will again realize that she has to save Buckbeak, and will do so much as described in the CD timeline. Then she will move to that spot from which she can effectively watch the Womping Willow, and see the Original group emerge. Seeing Harry confront Lupin, she will realize that he is about to be shredded, and will make the wolf call. After that, she flees from the werewolf and is rescued by Buckbeak.

Harry runs toward the lake to catch up with Sirius. Duplicate Hermione, however, now knows that Harry will die there, and once she is free of the werewolf she heads for the lake, emerging on the far side as she does in the film, but without Harry.

That is not what happened in the book. In the book, Harry goes to the far side of the shore by himself, trying to find his father, whom he believed cast the Patronus that saved him. That is why the book fails but the movie succeeds. In the movie, Harry and Hermione race to the shore together to seek to save Sirius; thus it is reasonable to suppose that when alone, Hermione also will race to the shore opposite Original Harry and Sirius, to attempt to save Harry from the dementors closing on them. This is the critical moment at which you have to believe something for which there is almost no evidence, or the film fails: Hermione must successfully protect Harry from the dementors, and she must do so from the opposite side of the lake.

That's not as entirely impossible as one might think. First, she has seen Harry cast the Patronus before--when Malfoy faked being a dementor at a Quidditch match to scare him. Besides, she is the smartest witch of her generation, so attempting to cast a spell she has seen done exactly once is not completely impossible. It has a non-verbal non-physical component, that the caster must think of a happy thought; but the ridikulus spell used against boggarts also has a non-verbal non-physical component, so it is possible that she might pick up the similarity here. Also, the real difficulty in most castings of the Patronus is that the caster must think something happy while having all happiness sucked out of him by a dementor. Hermione is across the lake, not subject to the dementors' effects, and so should find the spell easier in that context than it would be were she in the midst of the trouble. In any event, she must cast a patronus from the opposite side of the lake, and it must save Harry's life. Note that it does not have to do more than that, and given her determination to save Harry she might succeed in getting the dementors to leave him but take their targeted Sirius. After all, the dementors weren't really after Harry, and if they get Sirius they might be driven from Harry fairly easily.

It is not clear what Duplicate Hermione will do next. Probably she will remove the chain from Buckbeak and release him into the forest to fend for himself. There is no saving Sirius, because the dementors have already done their worst. She has to be back to the door outside medical before the clock stops striking, so probably she will be walking up the corridor toward a smiling Dumbledore with news that Harry is alive--news Dumbledore already knows, and which she will never deliver, because Original Hermione is now standing in Madame Pomfrey's rooms with Original Harry, turning the time turner to go back to save Sirius and Buckbeak. The EF timeline ends, and a new history is about to be created.

Back to top of page.


Saving Sirius
This time, Duplicate Hermione's information is different, and she has a companion, and it impacts how the new history plays.

As mentioned, Duplicate Hermione wonders, as she does in the film, why the Original trio are not leaving Hagrid's cottage. She then sees the stone that broke the pot, and also must recall that Harry was hit by some unexplained object, and thus that she must have thrown a stone or dirt clod at him. As mentioned, this is a wasted action, as the trio does not leave until the executioner is at the door anyway. She has Harry's help this time, and two working together move Buckbeak. They are, of course, unaware that Hermione was able to do this herself in a previous history.

Most of the events from there play out as we see in the film, with one exception. They circle around, using Hermione's wolf call to lure Lupin away from Harry, then flee through the forest, rescued by Buckbeak's intervention, then, because Harry wants to save Sirius, they rush out onto the far shore of the lake, where they see Original Harry and Sirius, and the approaching dementors.

Here, Duplicate Harry knows only that someone cast a patronus of some sort, and it saved him. It is likely that this patronus was not well-formed, and was not terribly powerful, but it did save him. He might be looking for who did it; however, he did not see himself do it, so he does not think it was his father. He emerges into the light, and when he sees that there is no one else, it does not occur to him (or to her) that Hermione must have done it. Thus he does it himself, casting the patronus powerfully enough to save himself and Sirius.

While Sirius and Original Harry are being moved to their respective locations, Duplicate Harry and Duplicate Hermione are off to the astronomy tower aboard Buckbeak. Once Sirius is locked up in the tower, they release him, and he leaves on the back of the hippogriff while they race back to the medical wing before the loop ends. Again, though, they do not make it. The GH timeline is not the end; they have to play through this act one more time.

Back to top of page.


Tying Up Loose Ends
There is one difference between the timeline just outlined as the third change (the GH timeline) and the one we see in the movie, and that is this: Harry saw himself, and thought he was seeing his father. That means that the timeline in the movie is one generation further along the sawtooth snap. Harry could not have survived the original AB history because even had Lupin not torn him to shreds, the dementors would have gotten him. Therefore he could not have traveled back with Hermione to create the CD timeline, and could not have rescued himself, and could not have seen himself rescue himself. Hermione had to rescue him there, in the EF timeline. Thus for Harry to have seen himself, he must have traveled back with Hermione after she rescues him from the dementors in the EF timeline, making that the GH timeline in which he saw himself; but then when he travels back, he is a slightly different Harry, one who thinks he saw his father cast a patronus from across the lake, and thus is looking for his father when he finds his Original self there. This, in turn, means that it is the IJ timeline when Harry, standing on that shore, realizes that he saw himself, and casts the patronus with confidence, knowing that it was he who did so last time. His argument that he could do it easily because he knew he had already done it makes some sense in that context.

This resolves all the anomalies. Harry and Hermione rescue Sirius and send him away on Buckbeak, and return to the medical wing. Their originals are just leaving via time turner to do what they, the duplicates, have just finished doing, and Dumbledore admits them to the room as it is vacated. Time continues, and the history we see in the film is the only history anyone knows. The departing Original Harry and Hermione are exactly identical in every way to who the returning Duplicate Harry and Hermione were three turns back when they left, so they will repeat history exactly as we have seen it.

I said I might owe J. K. Rowling an apology. It turns out that I do not. The book version of the story fails; Harry dies, victim of the dementors by the side of the lake, with no one to save him, and so can never make the trip from the future to save himself. It happens that in trying to simplify events in the movie, raise the action, and give Harry someone to whom to express his thoughts in that critical scene, the filmmakers blundered into a solution to the anomaly that works. I'm pleased to be able to resolve the movie version, but regret that the book version is not so easily fixed.
Donnie Darko is also interesting, because it has an extra timeline (from memory, there's normally 3, I think. Something like 1. History if there were no time travel at all. 2. The history we see - the character going forward in time, then back, then forward again. 3. The history of the rest of the universe with time travel - goes forward, time-traveller appears, stuff happens, time traveller disappears, history continues. Something like that).

This film was a surprise. I didn't hear a word about it when it went to theatres, but then when it hit video suddenly I was inundated with questions about it. My response was delayed; apparently the local Blockbuster lost both copies (one broken, one stolen) and was having difficulty replacing them. Finally it arrived, and finally I saw it, and now it's time to address it.
I must acknowledge the assistance of my second son, Kyler Young, who discussed the film with me immediately following the first viewing, and spotted several points before I did. The film does potentially resolve; but it requires a more significant suspension of disbelief than merely whether someone can travel in time. I invite you to consider this analysis carefully, and welcome any suggestions on how it might be otherwise resolved.


The Original History
Donald Darko, or Donnie, is an emotionally troubled boy in any event; however, in the timeline we see, someone is playing games with his mind, trying to bring about an outcome that is worse for him than his fate. There is an original timeline which is if not exactly happy considerably less disturbing than the one in the movie. From the ending of the movie, we can see that there is another timeline that is also less disturbing than the one we see; in fact, we resolve the story to four timelines, the last being labeled GH, but this one, the AB timeline, worth reconstructing.

In this timeline, nothing and no one comes from the future. This is an essential element of the time travel theory on this site: there must be an original history which leads up to the point of departure of any time traveler (including an inanimate one such as a jet engine) before it can depart, and it cannot arrive within the past of that timeline until, in terms of its own sequential existence, it has had the opportunity to depart. Thus in the AB timeline, Donnie Darko is asleep in bed, hears no voice, and faces no disaster. The jet engine does not arrive.

Frank does not exist in this history either--that is, the guy in the warped rabbit suit who pushes him to do all kinds of strange things, the first of which is to come out of his room to the golf course. That also means Donnie doesn't do any of the terrible things we see in the film. He is seeing a psychiatrist, Dr. Lillian Thurmond, and he did once burn down an empty house, and he certainly has emotional problems for which he is taking medication, but in the film Frank tells him to vandalize the school and to burn down the house of motivational speaker Jim Cunningham. Without Frank, he doesn't do these things.

It's also worth noting that it was because of the flooding of the school that they had the emergency PTA meeting; and that it was at the emergency PTA meeting that gym teacher and dance coach Kitty Farmer accused English teacher Karen Pomeroy of teaching pornography in the form of a Graeme Greene short story, The Destructors, a story in which children vandalize a house by breaking a water main and then burn down another house. It is from this incident that Karen Pomeroy is fired roughly one week later; thus, since the school was not vandalized and the meeting not held, there was no reason to fire Miss Pomeroy. She continues to teach.

However, she does not write cellar door on the blackboard before she leaves, so Donnie does not see it there.

She does introduce Donnie to Gretchen Ross, and their romance won't be too much different than seen, but that he won't abandon her in the theatre to go burn down that house, because Frank's not there to goad him into it.

That also means that the child pornography ring is not uncovered, Cunningham is not discredited, and Kitty Farmer is not required to appear in court to assist his defense. Thus Mrs. Farmer will fly with the girls to be on Star Search, and not Donnie's mother Rose. His little sister Samantha will be on that plane, though, when it comes back.

Now it starts to get difficult. Because Donnie's mother Rose is not on the plane, she's in the house; and that means that when older sister Elizabeth gets accepted to Harvard they don't have the big party. They probably do have a family celebration of sorts, but not the costume party we see; and certainly Frank is not invited. Remember, Frank is a real person, apparently a friend of Elizabeth, who wears a sick rabbit costume to the party. Now there's no party.

This does not impact Gretchen and her mom; Gretchen's mom still vanishes, Gretchen calls the police, she's told to go somewhere safe, so she goes to Donnie's house, where certainly Mom Darko would let her stay (although whatever happened in that upstairs bedroom won't have happened, and there won't be a phone call telling them that the dance team is on its way home).

Yet we now have a problem. Tomorrow morning near quarter of seven that plane is going to hit that disturbance, and it's going to break up. One engine from it is going to be thrown back through time twenty eight days, and crash into Donnie's room. If he's there, he's dead. The only reason for him not to be there is if Frank lures him out of the room; but Frank can only do this if he's been shot by Donnie already in the future. So somehow, in the AB timeline, Donnie has to be where Frank is, and has to shoot Frank. How does this happen?

Back to top of page.


Reconstructing the First Timeline
As crazy as it sounds, there is a way to get there offered by the film. You have to believe in destiny, or something like that, but it's there.

At some point, while watching football with his father, Donnie starts to see these blurry watery images extending from people's chests telling him where they are about to go. He also sees one coming from his chest, showing him where he is about to go. Later he again sees these at the party; by now he's probably mentioned them to his psychiatrist, and is wondering what they are. There is no party; but then, he might well see that image extending from his chest, or from Gretchens, or possibly from both.

Donnie has not given a thought to time travel, and has not seen nor read the book by Roberta Sparrow, better known in the film and the neighborhood as Grandma Death; he has no reason to think to go there. However, if this blurry image is destiny and he follows it, he could well wind up at the same place, with the girl, and encounter the burglars. In the timeline shown in the film, when someone, who turns out to be Gretchen, is approaching him, he puts his face inside the end of the destiny, and sees and hears things within it that he finds disturbing; if indeed there are fragments within that destiny, they might inspire him to leave the house with Gretchen, to see where it is leading. The other two friends are not here at all this time, because there was no party so they were not at the house. The fight progresses, the girl is thrown onto the ground, and the car, swerving to avoid good old Grandma Death, runs over the helpless girl.

This would be a most incredible coincidence, perhaps. After all, we've just suggested that even though there is no party at Darko's house, Frank is still out wearing that rabbit costume and driving the same road. However, it's not so unlikely as all that. Frank strikes us as a party guy. One of the reasons Elizabeth and Donnie thought they could get away with the party was that the town is on the tail end of a four-day "Halloween Festival" at the Middlesex Pavilion Mall. People are wearing costumes and wandering around as it is. Somewhere there's going to be another party, and Frank is going to be at it, and he's going to go for beer. That he's on the same road is not so unusual, since it seems to be out in the boondocks and thus must be the road to somewhere else; if he was headed that way when there was a party at Darko's, there's every reason to think that's where he goes for the beer. The only remarkable coincidence is that he's at that spot at near enough the same time to run over Gretchen. Accept that part, and the rest is rather predictable.

Would Donnie have shot him? Note that he has the gun not because Frank told him to get it but because he followed that destiny thing he could see. He has not been through as much trouble over the past month, no burnt house, no crashed jet engine, no vandalized school; but perhaps because of this he's a lot closer to Gretchen, and a lot more upset about this. Even without Frank's toying with him, he's got emotional problems. He shoots Frank in the eye, and kills him.

The Frank who appears to Donnie in the movie is a ghost--a supernatural being, the spirit of a man who died an unexpectedly violent death. The ghost has taken advantage of the portal to travel back in time to a moment just before the arrival of the jet engine, and has chosen to do so in order to ruin the life of the person who killed him. He might even be thinking that if he does it right, Donnie Darko will be incarcerated before the shooting, and he won't be dead.

This timeline isn't quite over. Although Donnie's mother is home, his sister Samantha is on that plane. Donnie probably runs, probably with the body of the dead Gretchen, probably in the family car. His sister is on the plane; the plane hits the vortex, and part of it--plus the Ghost of Frank--travels back four weeks. The police never catch Donnie, because history has just been altered, and there is no future for this timeline.

Back to top of page.


A Few Quibbles
The timing of events is not terribly clear despite regular updates on the screen. There are a couple points at which it doesn't quite match, but these are minor.

In particular, it appears that Donnie is in his bed at midnight at the beginning of October 2, 1988, and that a voice lures him out on a long walk to the golf course. We could debate whether it's necessarily the case that the jet engine didn't hit the house until after Donnie was asleep on the golf course; what is clear is that he was far enough from it that he didn't hear it happen. Thus it was after midnight. At that moment, Frank tells him that the world ends in twenty-eight days, six hours, forty-two minutes, and twelve seconds.


Twenty-eight days later, the clock strikes midnight and we are told that it is October 30, and that there's only six hours to go. Even if we assume it was only a ten minute walk to the golf course, that's closer to seven hours. Yet it is a minor point, and accuracy is not a critical factor in this story.

Someone might observe, and correctly, that Donnie doesn't see the strange images that I have chosen to call "destiny" (for lack of a clearer word) until after he has possession of the book in which such images are described. It's certainly possible that he has already read about those images prior to seeing them; it is equally possible that he has not.

Why does this matter? If we assume that he cannot see destinies until after he reads about them in the book, then in the AB timeline he could not have seen them. The sequence of events in the CD timeline (still to come in our analysis, but represented in the film) is that Frank mentions time travel, Donnie asks an unnamed teacher about it, the teacher gives him the book, then he sees destinies. If reading the book is necessary to seeing destinies, then he can't see them in the AB timeline, and he can't have the gun or go to meet Frank.

Yet whether or not he had already read about them in the CD timeline does not have to be relevant. He sees them. His teacher has also read the book, presumably, and, also presumably, does not see destinies. Apparently some people see them. In talking to Dr. Thurman, Donnie tells her that he sees these destinies, and has read about them in the book, perhaps implying that after he saw them he read about them. If so, then the fact that they're discussed in the book is not the cause of him seeing them, and he might well have seen them in the AB timeline, and followed them.

Back to top of page.


The Story Presented
The film shows us the events of the CD timeline. As it begins, a jet engine has just traveled back through time and is headed for Donnie Darko's bedroom. If it hits while he's in there, he's dead. It would be a simple N-jump in that case, as he has nothing to do with the time travel event, so in twenty-eight days the plane will again hit the portal and lose an engine to his bedroom.

Frank interferes. Being a ghost, he is able to move more quickly than physical objects. He lures Donnie out of that room and to the golf course far from it in enough time that Donnie is unaware of the crash on his room until the next day. It's unclear whether he's sleepwalking or following the voice exactly, but either way the ghost saves his life.

He is cognizant of the fact that the ghost saved his life. Under hypnosis he later tells his psychiatrist that he has to do what Frank says because Frank saved his life. Frank has nothing but malice for him, and saved him solely to destroy him.

It is in this timeline that all of the events of the movie occur. Donnie vandalizes his school and burns down Cunningham's house. The child pornography ring is uncovered, and Rose Darko has to be on the plane with her daughter. They have the party, and the time travel book is in Donnie's hands, pointing him to Grandma Death when he's trying to figure out what happens. Again he shoots Frank, and carries the body of the dead Gretchen back to the house, escaping in the car.

At this point, he realizes something of what has happened. He tells Frank's companion to tell everyone it's going to be O.K. Perhaps he knows that Frank saved his life to make it worse; he certainly knows that a lot of grief came to a lot of people, including himself, because he was not in his room when that jet engine hit it. He decides that it would have been better for everyone, and particularly for Gretchen, if he had died in that bedroom. Thus he somehow takes advantage of the vortex to change history. He sends himself back to the past, or he sends his decision back to the past. Whatever he sends, he persuades himself not to leave that bedroom. This creates the EF timeline.

Back to top of page.


What Happens If He Dies
Normally if you intentionally and successfully change the past, you create an infinity loop, because you now are not in a position to do so. However, in this case it appears Donnie Darko may have successfully done it without such a problem.

Remember that in the first history, the AB timeline, there was no jet engine and no Frank, and so Donnie was never in any danger, and he didn't act terribly crazy--but he did kill Frank. That meant that in the CD timeline Frank was around to push him into all the terrible things he hadn't done the first time, but he still killed Frank. We were very close to an N-jump with a very sad ending when Donnie used the same time travel event to carry his thoughts and intentions back to himself in the past. Those thoughts must have been to ignore the phantom voice and stay in bed. Now he dies in his bed.

Because he dies in his bed, doesn't that mean now that he can't send his intents back to himself, instructing himself to stay in bed? Yes, it does mean that; but now it's not necessary. In this timeline, Gretchen never meets Donnie; she sits next to some other cute boy in Karen Pomeroy's English class. Donnie doesn't vandalize the school or burn down the house, because he's already dead. There will be muted celebration of Elizabeth's acceptance at Harvard, and Samantha will travel to be on Star Search. Rose will stay home, undoubtedly upset over the loss of her son and not needed to chaperone the girls since Kitty Farmer doesn't have to be at Jim Cunningham's kiddie porn arraignment. Donnie will not take the gun from the closet, and will not shoot Frank. Thus Frank is not a ghost; and not being a ghost, he can't take advantage of that portal to go back and play with Donnie. Further he has no reason to do so.

Thus, at the beginning of the EF timeline, Donnie Darko hears the voice of Frank the Rabbit, but he also feels his own desire, conveyed to him from the future, to ignore it. He stays in that bed, laughing with the knowledge that the death he's about to meet is better than the misery of the month he would otherwise live, and dies; and in so doing he undoes both the rabbit and the desires. Both of them having been undone, we must take it once more, to the GH timeline.

In the GH timeline, Donnie Darko is in bed. No rabbit appears because at the end of the EF timeline Frank was not killed. No impression to stay where he is comes, but it doesn't matter because there's no reason for him to move. The airplane engine sails from the future to crash into his bedroom, and he dies.

Because he is dead, the rest of this timeline plays out as the EF timeline did. In the end, Frank is again not shot, Gretchen Ross is not hit by a car, Karen Pomeroy does not lose her job, Rose Darko is not on the plane, and Jim Cunningham's pornography ring is not exposed. Well, that, and that the that Samantha Darko is still killed in the plane crash, and that the NTSB's Bill Smith (see Millennium) is trying to figure out how it happened that a boy was killed by a jet engine that fell from a plane that exploded in midair twenty-eight days later on which his little sister happened to be a passenger, those are the sad points of the film. (What, do you think the NTSB isn't going to realize that the plane that crashed on October thirtieth is missing exactly the same sort of engine that crashed into the house on October second, the origin of which was never discovered?)

Back to top of page.


Conclusions
The fourth timeline is stable; all causes and effects exist in a single history. It is not the history we see in the movie; it is the history in which Donnie Darko is killed by a freak accident when the jet engine hits his bedroom while he sleeps. Time continues.

Frank said that it would be the end of the world; it was not the end of the world. It was some sort of temporal vortex or portal. It isn't the end of the world for Frank, who now is not shot. Donnie's sister Samantha is killed, and the other members of the Sparkle Dancers, along with that crazy gym teacher Kitty Farmer and others on the airplane, but airplane accidents do happen. Time has not ended; the world may continue, even for Frank. Was Frank lying? Let's say he was confused by what had happened. Since he used the portal to travel to the past, he can't know what happened after the plane crash. Never having seen such an event before, he guessed it to be the end of the world. We'll note that he was mistaken about that, even though we don't fully understand the portal itself.

Some will be disappointed. In order for this film to work, we had to introduce two concepts that are clearly supernatural in some sense: destiny and ghosts. I'll take a moment to defend these, not as realities, but as necessary components of the solution to the movie. That is, if you don't accept them, the movie is impossible; but if you do, the timeline can be unraveled as presented above.

It is clear that Donnie sees those odd ripples of where people are going before he has any reason to expect them. He has not spoken about time travel to the teacher, and so does not have the book. Frank has not told him anything about these things. He sees them. Now, if they are madness, that's what they are; but they do seem to be presented as working, as showing where someone is going. The problem with dumping this point is that there does not appear to be any other way to get Donnie to the scene of the crime. If he does not shoot Frank within a few hours before the crash, he dies, and the movie doesn't happen. Thus there must be a reason why he is there, and why he shoots Frank, and this is the most plausible. The alternative is an even more incredible coincidence of some sort.


As to the ghost, it's readily apparent that the wound revealed by Frank in the movie theatre is the wound inflicted on him by the gunshot after the accident that killed Gretchen. It is also apparent that the gunshot was fatal. Frank can't be doing this as a living person. There's no reason to think that the Frank who is alive twenty-eight days before the accident has the power or the knowledge to do the things he does, let alone the motive. Only the Frank Donnie shot has either the motive or the ability to do this; and only that Frank is stuck for all eternity in that stupid bunny suit, if theories about ghosts are correct. This has to be the Frank Donnie killed, and therefore he is a ghost from the future.

Now, I don't particularly believe or disbelieve in either ghosts or destiny. I'd say that ghosts strike me as the more plausible of the two, but both are doubtful enough and difficult enough to prove that it's foolish to pursue argument on the subject. Don't think I'm arguing for the existence of either. I'm only claiming that they are necessary for this movie to work as a time travel story.

Is it a time travel story? In any film as, shall we say, artistic as this, there will be uncertainties. I have read many interpretations of 12 Monkeys which maintain that that film has nothing to do with time travel. Like that one, it is entirely possible that this is not a portrayal of real events but of madness. It is possible that Donnie Darko imagines much of what happens, or does some of it and imagines other parts, that no jet engine hits his house and in the end after shooting someone he collapses into a catatonic state and believes he died four weeks before. It is as possible that this is a modern telling of Occurrence at Owl Creek Bridge (Ambrose Bierce), a story in which Donnie sees the means of his death break through the roof of his room and in the instant before it crushes him in his bed dreams his escape and all that would flow from it, only to have it end abruptly as the rope tightens around his neck. In such cases, it doesn't belong on this site at all, as this is about analyzing time travel and time travel movies. Those who have written to ask about the film apparently think it is a time travel movie, and I have approached it with that assumed. I'm certainly interested in other interpretations, but if it is a time travel movie, I want to know whether it works.

Given that, it does work, and with less difficulty than I'd anticipated, based on the initial mail.

Flickerdart
2009-11-24, 10:47 PM
The really fun thing is that the ring has to be invulnerable, or renovated with an object outside the loop. Else it will be worn down over time. Which means you can take the ring and use it for some physics breaking machines if you can pry it loose from the time loop.
Once in a while the gem cracks and has to be replaced, or the band wears out and needs changing. But it's still the same ring. :smallbiggrin:

Cespenar
2009-11-25, 02:41 AM
Then it sounds like you're just describing a rather needlessly obfuscated version of the "traveling back creates an alternate timeline splitting off from the point of your arrival" model.

Which is a perfectly valid model, but adding the complication that it's only your mind that's traveling back to hijack your old body adds nothing to its validity. To put it another way, the model would work just as well and contain exactly as many (zero) paradoxes if you traveled back in body as well as mind and could, in fact, shake hands with your past self.

It is actually very simple, but as I said, it's probably my descriptive skills that lack.

Once again should I try that it very closely resembles the save/load mechanism, which shouldn't be much of a hard thing to understand on its own, and at least doesn't include the utter stupidity (IMO) that you could shake hands with your past self.

kamikasei
2009-11-25, 03:11 AM
To be honest I'm pretty sure I'm understanding what you're saying - I think you're simply wrong.

Talking about "save/load" just brings us back to solipsism. The reason you can load an earlier save and erase the progress made since in a game is because you are a separate entity outside the game with continuity of memory and the game's "time" is an artificial thing. Such an analogy makes no sense when applied to the real universe.

Aside from not wanting people to be able to shake hands with their past selves - which however stupid you may consider it to be, is not necessarily paradoxical - how and why does your model have fewer paradoxes than if people could bodily travel back in time, but doing so put them in an alternate timeline, not their own past?

PirateMonk
2009-11-25, 11:03 AM
The really fun thing is that the ring has to be invulnerable, or renovated with an object outside the loop. Else it will be worn down over time. Which means you can take the ring and use it for some physics breaking machines if you can pry it loose from the time loop.

Actually, it works much better if it wears away. Eventually, it will break or become so ugly that your grandfather discards it. He then either buys the original or goes without a ring for a cycle, avoiding the paradox.

Cespenar
2009-11-25, 11:46 AM
Talking about "save/load" just brings us back to solipsism. The reason you can load an earlier save and erase the progress made since in a game is because you are a separate entity outside the game with continuity of memory and the game's "time" is an artificial thing.

Seeing that "you" as a person isn't of any importance other than being the one that uses the unique device of time traveling, I don't see how it would hurt if it's the machine that is the source of all the solipsism in my theory.


Such an analogy makes no sense when applied to the real universe.

Again, you're implying that all the other theories of time travel are approved science, and I'm the only one talking rubbish around here.


Aside from not wanting people to be able to shake hands with their past selves - which however stupid you may consider it to be, is not necessarily paradoxical - how and why does your model have fewer paradoxes than if people could bodily travel back in time, but doing so put them in an alternate timeline, not their own past?

First off, that's not time travel, that's dimensional travel. If I don't end up in my own timeline but travel into the past of an alternate timeline, I simply haven't really travelled to my past, which makes it not time travel.

Second off, alternate timelines is a separate theory. Your example tries to fortify the time travel theory with another unproven theory. Mine at least tries to be consistent on its own, despite how ridiculous it may sound. And the reason you're saying that the alternate timeline theory is not paradoxical is exactly this. It bases one unproven theory on another, which makes the whole deal so blurry that it can claim to be consistent, while that's just a logical fallacy.

Lord Seth
2009-11-25, 11:52 AM
This may sound a bit egotistical, but I liked one that I wrote.

What ultimately happens is one character goes back in time to tell himself not to do something that results in a really bad future. Aha, you ask, but how then, would he go back in time to warn himself if there was nothing to warn? What the character then does is use a time machine to go back in time and give himself the exact same message. It's actually a kind of cool combination of temporal paradox and stable time loop that averts the problems with both.

I think stories that are of the variety of having to go back in time after someone to stop the other people from messing up the timeline works okay as well, because once all is said and done all you've done is avoided a change in the timeline (or at least avoided a change significant enough to really notice). Like "The City on the Edge of Forever" in Star Trek or "Past Tense" from Star Trek: Deep Space Nine. Of course, the Star Trek universe is infamous for changing the rules of time travel whenever it pleases, but I liked the way those two were done, especially the former. (I also wrote a something of this variety)

PirateMonk
2009-11-25, 12:04 PM
First off, that's not time travel, that's dimensional travel. If I don't end up in my own timeline but travel into the past of an alternate timeline, I simply haven't really travelled to my past, which makes it not time travel.

Okay, if your theory doesn't involve alternate timelines, what's this about?


They happened in the "real" past. Which doesn't exist in the "normal" timeline. Real timeline > Normal timeline, to clarify. Real timeline goes forward no matter what, while normal timeline is changeable.

Cespenar
2009-11-25, 12:14 PM
@PirateMonk: That was to make things easier to understand. The only timeline/dimension/whatever there is the "real" one. The "normal" timeline actually doesn't exist, I say "timeline" there to note our misconception of it. Think of it like playing a game. It's the player's timeline that matters, the game is just a part of it.

Blayze
2009-11-25, 12:44 PM
This may sound a bit egotistical, but I liked one that I wrote.

What ultimately happens is one character goes back in time to tell himself not to do something that results in a really bad future. Aha, you ask, but how then, would he go back in time to warn himself if there was nothing to warn? What the character then does is use a time machine to go back in time and give himself the exact same message. It's actually a kind of cool combination of temporal paradox and stable time loop that averts the problems with both.

Wait... How far does he go back to give himself the second message? Given what he's trying to prevent, he can't give the message to a version of himself who hasn't yet gone back in time, because both versions -- middle and future -- won't exist any more once his past self changes the future.

kamikasei
2009-11-25, 12:47 PM
Cespenar, I can't make heads or tails of what you're trying to say, and find your criticisms of my responses bizarre, so I think I'd better leave it there.


Actually, it works much better if it wears away. Eventually, it will break or become so ugly that your grandfather discards it. He then either buys the original or goes without a ring for a cycle, avoiding the paradox.

Think carefully about what eventually means here. The paradox is that the ring only goes through one iteration from an outside observer's point of view, but an infinite number from its own. It can't accumulate wear on successive trips, because it only appears in the past once.

Consider this reworking: instead of a ring, the traveler brings back a letter from his grandfather, on which there are a series of marks (counting, four-and-diagonal, you know the sort). He hands it to his grandfather, instructs the grandfather to increase the count by one, and keep it to give to his grandson (who will become the traveler).

Now, setting aside the question of physical wear on the paper etc., what will be the number in the letter? Bear in mind that if the traveler is truly traveling to his own past then there can only be one number. The grandfather only opens the letter once and only adds a mark once.

This is why it's a paradox. It can't happen. And the same applies to any physical object - just replace the incrementing of the number with the physical changes any object will accrue over any length of time.

Dervag
2009-11-25, 12:49 PM
So here is the challenge. Can someone name any single story, comic, book, movie, or whatever; which contains time travel and is not left with gaping plot holes caused by said time travel in the first place?Sure.

"By His Bootstraps," by Robert Heinlein. A man travels forward in time to become ruler of a future society, then brings himself forward in time, so that he can kidnap himself, bring himself before himself, and tell himself to acquire the materials necessary to travel forward in time to become ruler of a future society. Perfect internal consistency, except for the minor problem of what appears to be an imperishable notebook (unless the character recopies the notebook on each run through the loop; I can't remember).
________


No it can't Time Travel will never ever work as tool to strengthen story it will always weaken it. Stuff about time travel can be good but they are good in spite of time travel never because of it.Citation needed.
______


This may sound a bit egotistical, but I liked one that I wrote.

What ultimately happens is one character goes back in time to tell himself not to do something that results in a really bad future. Aha, you ask, but how then, would he go back in time to warn himself if there was nothing to warn? What the character then does is use a time machine to go back in time and give himself the exact same message. It's actually a kind of cool combination of temporal paradox and stable time loop that averts the problems with both.This has been done; it's one of the classic time travel plots. "By His Bootstraps" (see above) is a more elaborate variant on the theme. So is "All You Zombies."


I think stories that are of the variety of having to go back in time after someone to stop the other people from messing up the timeline works okay as well, because once all is said and done all you've done is avoided a change in the timeline (or at least avoided a change significant enough to really notice).That raises the inverse of the usual problem: How did you detect the original messup? If someone travels into the past to assassinate George Washington, and I travel into the past to try to stop them, how do I know I need to stop them? If I come from a timeline in which they succeeded I will take their success for granted (as if they had gone back in time, impersonated John Wilkes Booth, and killed Lincoln; that would just be history to me, not time traveler intervention). If I come from a timeline in which they failed, they failed and I can ignore them, right?

This is the problem with "City on the Edge of Forever;" it allows McCoy to go into the past and change the timeline, and then have the original, unchanged Kirk and Spock go back and do the same. It raises the question: why were Kirk and Spock in the year 22XX not changed along with everything else as a result of McCoy's actions in the year 193X?
________


Not necessarily. Knowledge is only an assortment of data. When you travel to the past, what knowledge you carry isn't from the "future", it's knowledge of a possibility. The moment you travel back, that "back" becomes the new "now", and that old "future" becomes nonexistent.Yes, but causally speaking, where did the data come from? If I have a .pdf file detailing the rise of the Robot Army, then travel back in time and prevent the rise of the Robot Army, that .pdf file still had to be created by someone at some point. But since no Robot Army exists, the file was never created as a work of nonfiction. It could have been created as a work of fiction... but if it were fiction, why would anyone bother to send it into the past?

The problem is that insofar as "an assortment of data" carries any meaning, any actual knowledge, it reflects a reality... which you just pre-emptively destroyed by acting on the knowledge.

The time travel paradox created by sending information back into the past is very well known; it started with the play Oedipus Rex in ancient Greece, roughly 2500 years ago. In the play, an oracle tells a king and queen that their son is fated to kill his father and marry his mother (information about the future, sent into the past).

Understandably, they wish to avpid this horrible fate, and try to get rid of the child. Which sets in motion a chain of events that leads to Oedipus being adopted, never knowing his biological parents... and one day killing his father and marrying his mother.

This is the only way to resolve time travel plots based on the idea of a single timeline self-consistently: anything that goes into the past either has no effect, or winds up creating exactly the situation that originally created it.
________


And yeah, though I can see how you linked it with solipsism, it isn't actually that. And your "this isn't time travel" phrase intrigues me. Does time travel actually exist that I seem to be digressing from it?

Anyway, despite all these, you probably realize that what I'm trying to explain is simply a theory, and (probably) no more possible than the other time travel theories out there. Still, it makes more sense to me than that old Back to the Future style "you unmake yourself!" jazz.
________


They happened in the "real" past. Which doesn't exist in the "normal" timeline. Real timeline > Normal timeline, to clarify. Real timeline goes forward no matter what, while normal timeline is changeable. You may have another look at the last part of my previous post.

I'm sorry too, these theories verge between the metaphysical and the fictional, and my inability to describe them really doesn't help.Why is one timeline "real" and the other "not real?" Can we create a large number of timelines, or are there only two? Because if we can create a large number of timelines, then you've just reinvented the "multiple timelines" interpretation of time travel... which is a good interpretation, but not a new one.

Thing is, in multiple-timelines there is no single privileged timeline. The timeline you came from is no more and no less real than the one you arrive in. Which seems to contradict your idea of a "real" timeline being immutable. In theory, your own personal private "real" timeline could be invaded by foreign time travelers, who would then proceed to change it, and nothing would be wrong with this.


Seeing that "you" as a person isn't of any importance other than being the one that uses the unique device of time traveling, I don't see how it would hurt if it's the machine that is the source of all the solipsism in my theory.The first problem is the implication that the machine is in fact unique: I cannot build a duplicate of it in any universe. If I can build duplicates, we face the possibility that I am "saving and loading" my mental state in a universe which is itself not truly real, because it can be accessed by someone else's "save and load" time machine.
______


Again, you're implying that all the other theories of time travel are approved science, and I'm the only one talking rubbish around here.Actually, no. The implication is that the reality of the physical universe is approved science, not that other time travel models are approved science. The attacks against your model are based on its inconsistency with itself, and possibly with the observed universe, not with other time travel hypotheses such as Niven's self-consistency principle.


First off, that's not time travel, that's dimensional travel. If I don't end up in my own timeline but travel into the past of an alternate timeline, I simply haven't really travelled to my past, which makes it not time travel.I must respectfully disagree. Travelling into someone else's past remains time travel, just as travelling into someone else's house remains physical travel. You are travelling, and you move through time. Time. Travel. Time travel. Simple.
_______


Second off, alternate timelines is a separate theory. Your example tries to fortify the time travel theory with another unproven theory. Mine at least tries to be consistent on its own, despite how ridiculous it may sound.Unfortunately, it has problems of its own, whereas the multiple timelines model and the single self-consistent timeline model can both operate without problems.

The "single self-consistent" system, also known as the "Law of Conservation of History" or the "Self-Consistency Principle," does this by making all paradoxes impossible: anything you introduce into the past has known effects, namely the ones it already had in your own past. You can't screw up your own past because your past includes your own actions, so anything you do will create your past, not destroy it.

The "multiple timeline" system eliminates paradxoes by exporting them into someone else's timeline; you can't screw up your own past because you aren't in your own past. You're in someone else's.

Your system does not eliminate paradoxes, because you can still screw up your own past by traveling into it.
_______


@PirateMonk: That was to make things easier to understand. The only timeline/dimension/whatever there is the "real" one. The "normal" timeline actually doesn't exist, I say "timeline" there to note our misconception of it. Think of it like playing a game. It's the player's timeline that matters, the game is just a part of it.So how do I distinguish between "real" and "fake" timelines? Is there any experiment I can perform to determine whether the universe I exist in is real or fake? If not, how is the " fake" universe immune to causality problems like the ones that would arise from time travelers intervening in the "real" universe?

Lord Seth
2009-11-25, 12:58 PM
Wait... How far does he go back to give himself the second message? Given what he's trying to prevent, he can't give the message to a version of himself who hasn't yet gone back in time, because both versions -- middle and future -- won't exist any more once his past self changes the future.Okay, here's how it worked. Not sure I explained it well. (in the story itself, the character actually gives the explanation to another character, who doesn't understand it at all, causing the original character to complain that he went to all that work to avert a temporal paradox and no one even knows what he did)

Step 1: Character goes back in time and tells his past self "don't do this, it'll cause bad things. Oh, and here's how you time travel."
Step 2: Character doesn't do the thing that causes the problems, then uses time travel himself to go back to himself and give the exact same message that was given to him in Step 1.
Part 3: Repeat Step 2.

Essentially it's deliberately setting up a stable time loop. Sure, the reason he goes back in time is now a bit different (as he didn't actually witness the bad future) but he still gives the same message, causing the same events to happen. Does that make more sense?


This is the problem with "City on the Edge of Forever;" it allows McCoy to go into the past and change the timeline, and then have the original, unchanged Kirk and Spock go back and do the same. It raises the question: why were Kirk and Spock in the year 22XX not changed along with everything else as a result of McCoy's actions in the year 193X?If I remember correctly, the place with the time portal was somehow exempt from changes in the past. Everywhere else was changed, but not that location. Kirk and Spock were there, so they weren't changed. A bit hokey, but it's an explanation at least.

PirateMonk
2009-11-25, 01:03 PM
Think carefully about what eventually means here. The paradox is that the ring only goes through one iteration from an outside observer's point of view, but an infinite number from its own. It can't accumulate wear on successive trips, because it only appears in the past once.

Consider this reworking: instead of a ring, the traveler brings back a letter from his grandfather, on which there are a series of marks (counting, four-and-diagonal, you know the sort). He hands it to his grandfather, instructs the grandfather to increase the count by one, and keep it to give to his grandson (who will become the traveler).

Now, setting aside the question of physical wear on the paper etc., what will be the number in the letter? Bear in mind that if the traveler is truly traveling to his own past then there can only be one number. The grandfather only opens the letter once and only adds a mark once.

This is why it's a paradox. It can't happen. And the same applies to any physical object - just replace the incrementing of the number with the physical changes any object will accrue over any length of time.

By traveling back in time, whether with a ring or with a letter, the traveler is changing the past, and by extension the piece of the present which he brings back. When the grandfather marks the paper, this will cause the paper the grandson receives to have one more mark on it. It will retain the mark when he hands it to his grandfather, creating an infinite loop.

Joran
2009-11-25, 01:05 PM
This is the problem with "City on the Edge of Forever;" it allows McCoy to go into the past and change the timeline, and then have the original, unchanged Kirk and Spock go back and do the same. It raises the question: why were Kirk and Spock in the year 22XX not changed along with everything else as a result of McCoy's actions in the year 193X?

Spock and Kirk were on the Guardian of Forever planet, which apparently made them immune to the effects of McCoy's actions. I'm not sure what would happen if they tried to leave the planet.

Star Trek always had some kind of handwaving whenever they're dealing with time travel that screws up the present/future. Star Trek: First Contact had the Enterprise get caught in the Borg's ship's temporal wake, which made them immune to the changes in the past. In Voyager, they had an entire ship that was outside the space/time continuum; that was fun.

Nerd-o-rama
2009-11-25, 01:11 PM
Okay, here's how it worked. Not sure I explained it well. (in the story itself, the character actually gives the explanation to another character, who doesn't understand it at all, causing the original character to complain that he went to all that work to avert a temporal paradox and no one even knows what he did)

Step 1: Character goes back in time and tells his past self "don't do this, it'll cause bad things. Oh, and here's how you time travel."
Step 2: Character doesn't do the thing that causes the problems, then uses time travel himself to go back to himself and give the exact same message that was given to him in Step 1.
Part 3: Repeat Step 2.

Essentially it's deliberately setting up a stable time loop. Sure, the reason he goes back in time is now a bit different (as he didn't actually witness the bad future) but he still gives the same message, causing the same events to happen. Does that make more sense?This is pretty much the only way to do it and make "sense". Intentional Stable Time Loops are the best way to stabilize anomalies into what that time travel in film site calls "N-Jumps". They require the time traveler's past self to behave in a predictable way based on their future self's advice and follow it exactly, but it should work. Stargate SG-1's episode 1969 worked this way (not sure about other time travel episodes) in some respects, using a letter delivered from Future Hammond to Past Hammond.


If I remember correctly, the place with the time portal was somehow exempt from changes in the past. Everywhere else was changed, but not that location. Kirk and Spock were there, so they weren't changed. A bit hokey, but it's an explanation at least.This here is the easiest and most commonly assumed kludge for avoiding the Grandfather Paradox - people in and around the time travel device are somehow immune to the altered causality caused by using it. Same thing happened in Star Trek First Contact, allowing the Enterprise to follow the Borg back to AD 2063 instead of being instantly negated by the Borg's successful time travel shenanigans (which would also explain why no one's memories or gene pools were altered by both sides' shenanigans either). It even applies in Back to the Future, though a partial exception is made for totally negating your existence, good job Marty.

kamikasei
2009-11-25, 01:17 PM
By traveling back in time, whether with a ring or with a letter, the traveler is changing the past, and by extension the piece of the present which he brings back. When the grandfather marks the paper, this will cause the paper the grandson receives to have one more mark on it. It will retain the mark when he hands it to his grandfather, creating an infinite loop.

Yes... that's the paradox.

He's changing his own past, which means that the piece of paper he's carrying picks up an extra mark, but always had that mark, but OH CRAP UNIVERSE BROEK.


Essentially it's deliberately setting up a stable time loop. Sure, the reason he goes back in time is now a bit different (as he didn't actually witness the bad future) but he still gives the same message, causing the same events to happen. Does that make more sense?

This leaves us with the paradox that the loop has no cause. That's pretty much endemic to stable time loops, though (unless, I suppose, the device transports you through space as well as time, and puts your start and end points outside one another's light cones. Hmmm...).

Just going from what you say, in a stable time loop, it's invalid to say that the reason for his trip is "now" different. It is what it always was, and has to be so.

Lord Seth
2009-11-25, 01:28 PM
This leaves us with the paradox that the loop has no cause. That's pretty much endemic to stable time loops, though (unless, I suppose, the device transports you through space as well as time, and puts your start and end points outside one another's light cones. Hmmm...).

Just going from what you say, in a stable time loop, it's invalid to say that the reason for his trip is "now" different. It is what it always was, and has to be so.Why is it invalid to say it is "now" different? The way it works is convoluted, but it seems perfectly valid to me.

kamikasei
2009-11-25, 01:31 PM
Why is it invalid to say it is "now" different? The way it works is convoluted, but it seems perfectly valid to me.

Because there was no "before" to become "now". But perhaps I'm misunderstanding you - are you saying your story had a mutable timeline which in the story gets changed once to become a stable time loop? That's still paradoxical, since it means the cause of the events was removed from existence (and the loop set up does not fully cause itself).

To clarify: if it's a stable time loop, it has to be happening in a self-consistent, can't-change-the-past timeline. If it's an approximation to a stable time loop within a mutable timeline, then it's inevitably imperfect and continues to create paradox.

PirateMonk
2009-11-25, 01:32 PM
Yes... that's the paradox.

He's changing his own past, which means that the piece of paper he's carrying picks up an extra mark, but always had that mark, but OH CRAP UNIVERSE BROEK.

That's not a paradox. It's just a loop. If the marks do not in any way damage the paper or increase its mass or otherwise change it in an unintended way, nothing will happen except a paper with infinite marks is created. If the paper does change, then eventually it will be replaced or the loop will disintegrate, as I described for the ring.

Nerd-o-rama
2009-11-25, 01:34 PM
The idea of a paper with an infinite amount of marks/ink on it doesn't strike you as impossible?

kamikasei
2009-11-25, 01:34 PM
That's not a paradox. It's just a loop. If the marks do not in any way damage the paper or increase its mass or otherwise change it in an unintended way, nothing will happen except a paper with infinite marks is created. If the paper does change, then eventually it will be replaced or the loop will disintegrate, as I described for the ring.

"Eventually" is meaningless here. That's really all that can be said.

Lord Seth
2009-11-25, 01:42 PM
Because there was no "before" to become "now". But perhaps I'm misunderstanding you - are you saying your story had a mutable timeline which in the story gets changed once to become a stable time loop? That's still paradoxical, since it means the cause of the events was removed from existence (and the loop set up does not fully cause itself).

To clarify: if it's a stable time loop, it has to be happening in a self-consistent, can't-change-the-past timeline. If it's an approximation to a stable time loop within a mutable timeline, then it's inevitably imperfect and continues to create paradox.What happens is that the original cause (knowing what the future is like and warning your past self) is replaced by the new cause (your future self having previously told you the message, and then going back in time to in turn, tell yourself). The original cause is replaced by a new cause, which forms the stable time loop.

Or, let's not even look at it under the idea of a stable time loop and look at it under the temporal paradox; that is, the idea if someone from the future tells you something, and you do something to avert that future, then how the heck did they tell you if the future didn't happen? The reason here is that you still communicated with your past self, because in the replacement timeline you still had a reason to.

(I really get the feeling I'm not explaining this well...)

Mewtarthio
2009-11-25, 01:43 PM
Yes... that's the paradox.

He's changing his own past, which means that the piece of paper he's carrying picks up an extra mark, but always had that mark, but OH CRAP UNIVERSE BROEK.

Grandfather receives a paper with an arbitrary number of marks (say, five). Traveler walks off, chuckling about how he's just torn causality a new one. Grandfather scratches his head and discards the paper. Ten years later, traveler takes a paper with five marks and goes back in time to meet his grandfather.

PARADOX AVERTED!

PirateMonk
2009-11-25, 01:47 PM
The idea of a paper with an infinite amount of marks/ink on it doesn't strike you as impossible?

Ink adds to its mass. As ink is added, assuming the paper somehow survives, it will eventually become too heavy to move.

If purely superficial "marks" are used instead, then an infinite number is entirely possible, though the grandfather may decide the exercise is pointless once the paper is completely covered.


"Eventually" is meaningless here. That's really all that can be said.

How so?

kamikasei
2009-11-25, 02:01 PM
How so?

Because the fact that there's only one timeline, yet the object traveling is experiencing an infinite loop, is precisely the paradox.


(I really get the feeling I'm not explaining this well...)

You're explaining it fine. However there are two ways it can go and each has a problem.

One: it's a stable time loop in a self-consistent timeline in which case nothing happens to cause it, it just springs in to being inexplicably. That's weird.

Two: the timeline can be changed, and at some point there is a robot war that someone travels back from. Let's say his past self is A, and the guy traveling back from the robot war is A'. And he tells A to set up the loop. So A travels back in time, becoming A''.

When does A'' arrive? After or at the same time as A'? Then the presence of A' in the past still has to be explained - he's coming from outside the loop. Before A'? Then he's not recreating the loop - he's gone to a different starting point. The slightly-more-past A he speaks to will not have ever met A', so when he goes back in time to become A''', he's not A'', and essentially the loop has to precess ever further back until the entire universe is consumed in flames and horror. And since there's no external observer to perceive these iterations, all of them have to happen at once, which means the loop can never stabilize.

Lord Seth
2009-11-25, 02:34 PM
One: it's a stable time loop in a self-consistent timeline in which case nothing happens to cause it, it just springs in to being inexplicably. That's weird.Something did happen to cause it. It was replaced with the dependent cause, sure, but the original independent cause was only necessary to start the loop, not to continue it.


Two: the timeline can be changed, and at some point there is a robot war that someone travels back from. Let's say his past self is A, and the guy traveling back from the robot war is A'. And he tells A to set up the loop. So A travels back in time, becoming A''.

When does A'' arrive? After or at the same time as A'? Then the presence of A' in the past still has to be explained - he's coming from outside the loop. Before A'? Then he's not recreating the loop - he's gone to a different starting point. The slightly-more-past A he speaks to will not have ever met A', so when he goes back in time to become A''', he's not A'', and essentially the loop has to precess ever further back until the entire universe is consumed in flames and horror. And since there's no external observer to perceive these iterations, all of them have to happen at once, which means the loop can never stabilize.What happens is that A' is replaced by A. No further iterations, no consumption of the universe.

kamikasei
2009-11-25, 02:43 PM
What happens is that A' is replaced by A. No further iterations, no consumption of the universe.

I don't see how that can be so. A sees A' and deals with him, and A'' is not A'. So A'' cannot create the encounter he remembers by traveling back himself. He can only create a new encounter between A and A'' - and so on down the chain. None of them can quite play the part of the person they remember encountering, which is necessary to get a stable loop.

Lord Seth
2009-11-25, 02:48 PM
I don't see how that can be so. A sees A' and deals with him, and A'' is not A'. So A'' cannot create the encounter he remembers by traveling back himself. He can only create a new encounter between A and A'' - and so on down the chain. None of them can quite play the part of the person they remember encountering, which is necessary to get a stable loop.Why couldn't they play the part of the person they remember encountering? Say the exact same lines, do the exact same thing.

kamikasei
2009-11-25, 03:09 PM
Why couldn't they play the part of the person they remember encountering? Say the exact same lines, do the exact same thing.

Because the world is very, very complicated. Or even more shortly, chaos theory.

Cespenar
2009-11-25, 03:50 PM
@kamikasei: At least I agree on that (to end this). Based on your responses, it's most likely that I have problem converting my thoughts into words, so I'll not push further.

@Dervag: Sorry to let you down after your elaborate reply, but as I said, I don't feel like I'm making myself clear when explaining my theory, so further discussion will be pointless.

This is not to imply that my so-called theory was flawless, but... anyway. Signing out from the thread.

Lord Seth
2009-11-25, 03:58 PM
Because the world is very, very complicated. Or even more shortly, chaos theory.And how, pray tell, does that apply here? Same conversation, same sequence of events, same everything. It turns into a stable time loop. Explain what would go wrong that would render it to not work.

Unless you're arguing that under "chaos theory" a stable time loop is innately impossible, which seems irrelevant as this situation assumes a stable time loop is possible. Kind of like giving an argument that time travel is impossible in response to this thread.

Talya
2009-11-25, 04:14 PM
The Many Worlds Interpretation (http://en.wikipedia.org/wiki/Many-worlds_interpretation) insists there are infinite alternate timelines in existence even without Time Travel. If one subscribes to it (as much of science does, since the only real alternative is a deterministic universe) it makes the paradoxes created by altering the past far easier to take.

Soras Teva Gee
2009-11-25, 04:26 PM
I see a couple of errors people are making regarding time loops.

Namely that the time traveler 'sets up' the time loop. This error neglects that the time traveler himself is the original cause and a true stable time loop is not a cycle of iterations but a circle in time, its begining and its end have no meaning. There is no original uncaused cause because the cause creates itself in the first place,

The beginning of this events is no more meaningful then the beginning or end of a mobius strip.

Anything involving iterations misses the problem of what you are iterating in. An object sent through a true stable time loop only appears in the past once and only is sent back once. You end up with the same object occupying the same space (latoms on atoms) at the same time. To cycle the object by adding marks you must somehow erase the 'old' iternation with -1 mark, yet retain those events as somehow having occured within the relative history of the object.

The easy way out is a sort of universal denial, the object is never put into the loop that way to begin with. The story is never written and never occurs in the most fundamental way imaginable, trying will fail the same as changing the past will fail because the attempt to change it was the original cause and we are taken from cycle to circle.

The trick is this does not disallow time travel or a non linear chain of events. You send your grandpa's ring back and he looses it only to later buy a 'replacement' and you dig up the 'original' ring sent back from the park grandpappy lost it in. The ring goes through extra time but ultimately has one (and only one) personally linear chain of events that is non-linear to an outside observer, as the 'replacement' is the 'original' at an early point. In this solution as applied to the paper, grandfather never marks the paper but keeps it for some reason, so the loop remains unwritten. (And you have below)

For the ring problem it can remain a static object and it simply has no origin, whatever that means. I feel the urge to make up the term 'quantum ether' to describe this, it comes from meaningless nothingness at a fundamental level. All that really suffers in this is our vain assumption the universe has to make sense

Alternately you can you can solve the iterations of the time loop by supposing that attempting to send a paper with X+Y marks on it back can only be done in the X iteration. The paper only arrives in the past with X marks on it and the directions to put +Y marks on it. Where the paper with X+Y marks goes I really don't know, probably into literal nothing.

Ultimately otherwise you have to create an infinite number of rings or marks in a finite span of time, which 'inevitably' (or rather instantly yet infinitely) cause a change to past when the loop breaks down. And changing the past is just a traditional time paradox. I've always voted for the "OH SHI..." resolution to those but seeing as we still exist one hasn't happened.

kamikasei
2009-11-25, 04:33 PM
And how, pray tell, does that apply here? Same conversation, same sequence of events, same everything. It turns into a stable time loop. Explain what would go wrong that would render it to not work.


Because "you from the past, gone a little way forward in time and traveling back" and "you from the post-robot-war future" are extremely different entities and every way in which every particle of your bodies differs is a way that the conversation differs. It doesn't matter if the transcript would read the same. It is not the same conversation, it is not the same sequence of events, it is not the same everything.

Lord Seth
2009-11-25, 05:04 PM
Because "you from the past, gone a little way forward in time and traveling back" and "you from the post-robot-war future" are extremely different entities and every way in which every particle of your bodies differs is a way that the conversation differs. It doesn't matter if the transcript would read the same. It is not the same conversation, it is not the same sequence of events, it is not the same everything.But all that matters is that the results are the same. The result is using the time machine to go back in time and warn yourself. How does any minute difference in particles going to change that?

Furthermore, I should probably clarify a few things. To summarize the story, the character ends up being brought into the future, a bunch of stuff happens, then he goes back and warns himself. This is--for him, anyway--a matter of hours. Once his past self is warned, said past sense, in a few hours, goes back and gives the exact same message to his past self. How would anything be changed? Especially given that said character is extremely intelligent and would know to have the exact same conversation in the exact same way.

Mewtarthio
2009-11-25, 05:21 PM
The Many Worlds Interpretation (http://en.wikipedia.org/wiki/Many-worlds_interpretation) insists there are infinite alternate timelines in existence even without Time Travel. If one subscribes to it (as much of science does, since the only real alternative is a deterministic universe) it makes the paradoxes created by altering the past far easier to take.

And just how is a deterministic universe any more problematic than the many-worlds theory?

Talya
2009-11-25, 05:29 PM
And just how is a deterministic universe any more problematic than the many-worlds theory?

I honestly don't know. I tend to believe in a deterministic universe myself, but a lot of scientists hate the idea.

Devils_Advocate
2009-11-25, 05:37 PM
I must respectfully disagree. Travelling into someone else's past remains time travel, just as travelling into someone else's house remains physical travel. You are travelling, and you move through time. Time. Travel. Time travel. Simple.
But I'm traveling into the future's past right now. One day from now, I'll be one thousand years in the past of the time one thousand years and one day from the present. I am approaching this point in our decedents' past by traveling through time at a rate of one second per second.

It's a bit misleading to refer to this perfectly mundane sort of occurrence as "time travel".


Same conversation, same sequence of events, same everything.
Except that they're not the same sequence of events. In one sequence of events, a man is warned by a future version of himself who remembers a robot war he was never warned about and deciding to go back in time to warn his past self about it. In the other sequence of events, the same man is warned by a future version of himself who remembers receiving the warning of the robot war and averting it, and deciding to go back in time to give himself that warning. There are two different future versions of the man, with different mental states. A' =/= A'', notwithstanding that A can't discern which one he interacts with.

Or are you saying that somehow A' == A'', despite them having mutually exclusive properties? Or does A actually give himself false memories, in order to make A' == A''? Even if he does, that doesn't seem to make it any less of a classic stable time loop, since the actual cause of those memories wasn't the occurrence of the remembered events...

Hmmm. Let me put it this way: Does the robot war future actually happen, or doesn't it? If it does happen, then you've got multiple timelines. If it doesn't, then you would seem to be describing the present being influenced by a non-existent future that could have happened but won't. Since if the present is only effected by its real future, then... well, that's a perfectly "ordinary" stable time loop.

Cespenar also seems to be describing the present being effected by a future that won't happen (and for some reason labeling the won't-happen future "Real" in one description :smallconfused:). The difference is that you're saying that there's a replacement cause in the real future.

This gets into metaphysical semantics. In short: One can theoretically define "exist" such that something that doesn't exist can effect things that do exist. But, great gods, why would you want to?

Basically, the problem here is that the whole "one timeline that gets overwritten" description is essentially meaningless gobbledygook. If two mutually exclusive sequences of events happen over some period of time, that's two separate timelines. If you describe one sequence of events as effecting the other despite not happening, then you're simply describing things in a manner completely contrary to common sense, and still failing to actually explain anything. If there's only one sequence of events that's only effected by itself, then there is no overwritten timeline.

Lord Seth
2009-11-25, 06:27 PM
The difference is that you're saying that there's a replacement cause in the real future.

This gets into metaphysical semantics. In short: One can theoretically define "exist" such that something that doesn't exist can effect things that do exist. But, great gods, why would you want to?Brief note: You're using effect wrong. You should be using affect, because affect is an action.

I'm not sure what the point you're trying to make here is. Where'd you get this whole thing about that things that don't exist affects things that do exist? The original "warner" existed, affected something that existed, then stopped existing, and what he was replaced by did exist (or, depending on how you view it, he became the new warner). I don't see what your point is.

Devils_Advocate
2009-11-25, 07:03 PM
Brief note: You're using effect wrong. You should be using affect, because affect is an action.
Whoops. My bad.


I'm not sure what the point you're trying to make here is. Where'd you get this whole thing about that things that don't exist affects things that do exist?
It's one possible interpretation of the description you've given.


The original "warner" existed, affected something that existed, then stopped existing, and what he was replaced by did exist (or, depending on how you view it, he became the new warner). I don't see what your point is.
At what times did the original warner exist? At what time did he stop existing?

Did he also not exist at those same times? If so, you've got multiple timelines, which is one of my main points.

Are you positing a scenario in which A' and A'' somehow occupy the same space at the same time in a single timeline, despite being non-identical (by which I mean that they have different properties)? If so, I would suggest that describing A'' as "replacing" A' doesn't really get this across.

Let's suppose that A has his conversation with his future self at 1:00 PM to 2:00 PM on November 12, 2008. During this hour, does he converse with two different individuals (or, if you prefer, two different versions of the same individual), even though it seems to him that he's only talking to one?

On the other hand, if he only talks to one, what does that one remember as being the reason for traveling into the past to warn himself?

Dervag
2009-11-25, 09:13 PM
Think carefully about what eventually means here. The paradox is that the ring only goes through one iteration from an outside observer's point of view, but an infinite number from its own. It can't accumulate wear on successive trips, because it only appears in the past once.

Consider this reworking: instead of a ring, the traveler brings back a letter from his grandfather, on which there are a series of marks (counting, four-and-diagonal, you know the sort). He hands it to his grandfather, instructs the grandfather to increase the count by one, and keep it to give to his grandson (who will become the traveler).

Now, setting aside the question of physical wear on the paper etc., what will be the number in the letter? Bear in mind that if the traveler is truly traveling to his own past then there can only be one number. The grandfather only opens the letter once and only adds a mark once.

This is why it's a paradox. It can't happen. And the same applies to any physical object - just replace the incrementing of the number with the physical changes any object will accrue over any length of time.For a normal physical object, we can hypothetically have the ring repaired to its original state in each loop, preventing the paradox. In the case of the letter, there is a paradox unless we go to multiple timelines (in which case each timeline has a different, unique number of tally marks on the letter). And so we get an infinite temporal spiral, not an infinite loop.
_________


By traveling back in time, whether with a ring or with a letter, the traveler is changing the past, and by extension the piece of the present which he brings back. When the grandfather marks the paper, this will cause the paper the grandson receives to have one more mark on it. It will retain the mark when he hands it to his grandfather, creating an infinite loop.But an infinite loop cannot occur in a single timeline, hence the problem. In any one universe, the grandfather must receive one piece of paper with one number on it. He cannot receive an infinite number of pieces of paper.
_________


This leaves us with the paradox that the loop has no cause. That's pretty much endemic to stable time loops, though (unless, I suppose, the device transports you through space as well as time, and puts your start and end points outside one another's light cones. Hmmm...).You can still have stable time loops, as long as the system is restored to its initial state each time. There is a confusion (an event which need not have a cause, at least not in a one or two-dimensional timeframe), but no paradox (there is no direct internal contradiction among the facts of the scenario).
_________


That's not a paradox. It's just a loop. If the marks do not in any way damage the paper or increase its mass or otherwise change it in an unintended way, nothing will happen except a paper with infinite marks is created. If the paper does change, then eventually it will be replaced or the loop will disintegrate, as I described for the ring.This still fails for a single-universe model. In multiple timelines, there is no loop, just a spiral that looks like a loop if you try to superimpose all the myriad timelines. But in a single-universe setting, we are left with the contradiction that my grandfather receives a letter with one mark and a letter with two marks and a letter with three marks and... and so on... but still only receives one letter.
_________


"Eventually" is meaningless here. That's really all that can be said."Eventually" is meaningful in the letter's frame of reference, but not in the universe's. The letter perceives linear time, as it is handed back and forth between an endless succession of identical grandfathers and grandchildren. But in the abstract, you're right, "eventually" is not a helpful way to describe a single-universe stable time loop. In the letter's frame of reference it will ultimately get old and yellow and fall apart, and I think that's what Pirate Monk is getting at.

But since many "stable time loop" stories presuppose the ability to keep going round indefinitely ("All You Zombies"), this is not helping.
_________


Grandfather receives a paper with an arbitrary number of marks (say, five). Traveler walks off, chuckling about how he's just torn causality a new one. Grandfather scratches his head and discards the paper. Ten years later, traveler takes a paper with five marks and goes back in time to meet his grandfather.

PARADOX AVERTED!And that's the other way to solve the problem: invoke the Law of Conservation of History. Which, if we restrict ourselves to single-timeline scenarios, is my preference, though I can stand a temporal loop so long as it doesn't raise the issue of the object(s) in the loop eventually falling apart.
_________


You're explaining it fine. However there are two ways it can go and each has a problem.

One: it's a stable time loop in a self-consistent timeline in which case nothing happens to cause it, it just springs in to being inexplicably. That's weird.

Two: the timeline can be changed, and at some point there is a robot war that someone travels back from. Let's say his past self is A, and the guy traveling back from the robot war is A'. And he tells A to set up the loop. So A travels back in time, becoming A''.

When does A'' arrive? After or at the same time as A'? Then the presence of A' in the past still has to be explained - he's coming from outside the loop. Before A'? Then he's not recreating the loop - he's gone to a different starting point. The slightly-more-past A he speaks to will not have ever met A', so when he goes back in time to become A''', he's not A'', and essentially the loop has to precess ever further back until the entire universe is consumed in flames and horror. And since there's no external observer to perceive these iterations, all of them have to happen at once, which means the loop can never stabilize.The trick here is that A'' must BE A', being identical to him, and arriving at the same time. There never was an A' in a single-timeframe picture, because A' came from a different universe that is outside the picture, one that "did not" happen for a given value of "did not."

A'' need not be identical to A' down to the atomic level, but A'' needs to be functionally identical to A', in the only way that matters: A will react to A'' as he would to A', avert the robot war, and eventually become A'' himself. A' was an illusion, and never existed.

Thus, stable loop. There is merely a man informing himself what he needs to do to avert the robot war (which includes informing himself what he needs to do to avert the robot war (which includes informing himself what he needs to do to avert the robot war (...))). Weird, but stable.

========


But I'm traveling into the future's past right now. One day from now, I'll be one thousand years in the past of the time one thousand years and one day from the present. I am approaching this point in our decedents' past by traveling through time at a rate of one second per second.

It's a bit misleading to refer to this perfectly mundane sort of occurrence as "time travel".Yes, because it doesn't involve any violation of the natural behavior of time or other extraspatial dimensions. Which multiple-timeline time travel does. It's hardly mundane.


On the other hand, if he only talks to one, what does that one remember as being the reason for traveling into the past to warn himself?He remembers traveling into the past to warn himself to avert the robot war (which requires traveling into the past to warn himself to avert the robot war (which requires traveling into the past to warn himself to avert the robot war (...))). Fortunately, past-him takes future-his word for it, and does what future-he requires. The world at large may never know what a nasty robotic bullet they dodged.

You could even make a movie about it. Something about... termination.

Mewtarthio
2009-11-25, 10:38 PM
You could even make a movie about it. Something about... termination.

Ouch. You probably shouldn't bring that movie up in a time travel thread. It's just plain crazy. The first movie has a stable time loop, the second movie switches over to alternate timelines, the third movie contains bald-faced fatalism, and the fourth movie somehow manages to combine everything into a convoluted mess.

Yeah, fun mindless action (for the first two movies anyway), but not something you want to try and analyze.

chiasaur11
2009-11-25, 10:43 PM
Ouch. You probably shouldn't bring that movie up in a time travel thread. It's just plain crazy. The first movie has a stable time loop, the second movie switches over to alternate timelines, the third movie contains bald-faced fatalism, and the fourth movie somehow manages to combine everything into a convoluted mess.

Yeah, fun mindless action (for the first two movies anyway), but not something you want to try and analyze.

The third had inconsistent, inexplicable fatalism, from what I've read. Which, you know, really bugs me.

I mean, fine. You can't change the past. As valid a model as any other, I suppose. But Skynet is changing the past. In fact, the whole plot of the movie is Skynet changing the past.

I swear, if I was a computer from Star Trek, well, I'd be exploding right now.

Serpentine
2009-11-25, 10:54 PM
Ouch. You probably shouldn't bring that movie up in a time travel thread. It's just plain crazy. The first movie has a stable time loop, the second movie switches over to alternate timelines, the third movie contains bald-faced fatalism, and the fourth movie somehow manages to combine everything into a convoluted mess.

Yeah, fun mindless action (for the first two movies anyway), but not something you want to try and analyze.*ahem*

"Temporal Anomalies in Time Travel Movies
unravels
Terminator"
One of the most interesting temporal anomalies in fiction is that set up by the Terminator movies. Given the excessive number of trips through time which are made, we would expect the entire timeline to totally incohere and become impossible; yet with a few simple extrapolations, it all fits together. If we can find the beginning, we can trace the thread of time as it weaves forth and back. So we consider the timelines of Terminator.

Original History
The first movie presents a story in which a robot from the future comes back in time to kill the mother of an as-yet-unborn rebel leader. During the course of the movie, the child is conceived, as son of a man he has sent back from the future to protect his mother; the robot is destroyed and the father killed.

The patent absurdity of this immediately presents itself: the birth of the rebel leader is dependent upon his adult self sending his father back in time to meet his mother. Most reasoning adults would immediately write this off as impossible. However, in studying the patterns of temporal anomalies, we discover a form elsewhere referred to as an "N-Jump", which, properly understood, resolves this difficulty.

In an N-Jump, time extending from the past reaches point A, the point in time to which a traveler from the future will return, and beyond to point B, the point from which the traveler leaves for the past. During this segment of the time line, no changes have been made; it is the original unaltered sequence of events. When our traveler leaves point B, that time line ends--the history based on the A-B segment cannot progress, because the instant the traveler reaches point A, it is changed by his presence, and is re-named point C; this creates an alternate C-D timeline, with D being the same point in time as B. If at point D, the traveler can and does return to point C with the same intentions, history is able to continue into the future. This is an N-Jump.

(The temporal anomalies mentioned here are defined and described more completely in Appendix 11: Temporal Anomalies, of the Referee's Rules of Multiverser, an RPG from Valdron Inc, and in the Primer on Time on this site.)

If upon reaching point D, there is any reason why the traveler cannot return to point C, or if there is no reason for him to do so, the result is an infinity loop: the original A-B segment is restored, ending with the return to the C-D segment, reverting to the A-B segment in a perpetual cycle. In this case, there is no future beyond the time represented by points B and D.

The N-Jump provides the solution to the absurdity of the first Terminator movie. It suggests that this is not the original timeline, but the altered timeline. Although the story of the original timeline has not been told, a substantial amount of it can be reconstructed.

In the original timeline, Sarah Conner's life was fairly ordinary. Very near point A, she met a guy unknown to the altered timeline; he is the necessary original father of John Conner. So little remains of this timeline that almost anything is possible. John Conner might have had a different name, if his mother married. He might in this original timeline have been a girl. One thing is certain: when the war came, Sarah Conner's child was a thorn in the flesh of Skynet.

Of course, Skynet also must have been created in the original timeline, or there would be no war. However, the details of this creation are unknown, as will become clear as the thread unravels.

Back to top of page.


First Change: Sky Net
Sarah Conner's child is a major problem for Skynet, so the computer devises the ill-considered plan of sending a machine into the past to destroy Sarah Conner before the child is born. It is important to recognize that this is point B; nothing done after this moment follows from the same history. That gives us information about the C-D timeline, because we know that certain things did not happen. Of first importance, we know that the birth of Sarah Conner's child still occurred, because had that not happened, no one would have been sent back to protect her. In fact, had the Terminator successfully prevented the birth of that child, then the Terminator would not have been sent back (at point D), and the original timeline would have been restored, forcing the world into an infinity loop. However, something must have happened which John Conner needed to change; otherwise, no one would have come back to protect Sarah. The fact that John sends Reese tells us first that the Terminator failed, but second that something was dreadfully wrong.

This is a difficult point. John must know that the Terminator failed, or he would not be alive. Sending Reese is a calculated risk; if it goes wrong, John may cease to exist. However, there is a possible reconstruction of this segment which solves the problem. Remember that when the Terminator reached the past, he began killing people named Sarah Conner. Perhaps Sarah became aware of this, and fled. It is still necessary that she have a child. It may or may not have the same father as the other. The Terminator, intent on its mission, pursues her, catches her, and kills her. Its simple logic tells it the mission is accomplished, and it finds a place to shut down. It does not understand the basic failure: John Conner has been born. The first N-Jump resolves, and there is a future.

Back to top of page.


Second Change: Rebels
The grown Conner child doesn't like that future. He wants to save, not himself, but his mother. So he sends Reese back to protect his mother. Reese doesn't realize that the Terminator had to fail in order for this time line to exist; he thinks he's there to save Sarah's child. However, his presence drastically changes the entire timeline in several ways. First, he interferes with the birth of Sarah Conner's child, replacing the original father with himself. Second, he gives information to Sarah which she uses to protect herself and to prepare John to be a more formidable opponent. Third, he jump-starts the Skynet program.

This third point will require some reflection. In the second film, we are introduced to Cyberdyne Systems, the company which created Skynet by studying parts from Terminator. However, in the original timeline, someone created Skynet, and it was most likely someone else, someone who was very near the technological breakthrough when, in this timeline, the system goes on line. So now the dates are all shifted earlier, and the company credited with the development has changed.

This could create a minor loop. After all, it appears that the technology for Skynet is created sooner, preventing it from being created by someone else later. However, this is not insoluble if we recognize the creation of a brief sawtooth snap. The Terminators sent back by the original SkyNet may have been created by someone else, but as Cyberdyne takes over that technology those in the next timeline are built by them and sent back at the same point in time to do the same tasks, but with a different memory of their constructions.

Reese succeeds in saving Sarah's life. Normally, this would create an infinity loop, because the reason for sending Reese back is gone, so he wouldn't be sent back. But this new history gives birth to the John Conner we know, and he knows from his mother that Reese is his father, and that he saved his mother's life, so this John has a different reason to send Reese back. It is this segment which is shown in the first movie, and which forms the history for the second.

(This actually creates another sawtooth snap. John will have given Reese information which Reese will give Sarah, who will give it to John. This timeline may repeat several times until the information stabilizes--that is, what John tells Reese affects what Sarah tells John, and vice versa, so that information may be in flux for several repeats of the events, changing history slightly but not critically.)

Back to top of page.


Third Change: Skynet
This timeline still brings the war, and John and Skynet are still primary combatants. The computer, still not comprehending the hazards of attempting to change history, sends a more advanced Terminator back, this time to kill John. Remember: that instantly changes history. No one can remember a past in which that Terminator did not return. There is therefore an altered time segment again lost to us in which the T-1000 tried to kill John Conner and failed, but which John Conner felt it vital to alter by sending the first Terminator model back to protect himself from a machine which he knew from his perspective had already failed in its mission. Let's reconstruct it.

As unlikely as it seems, John Conner must manage to evade the T-1000 with the aid of such friends as he can find. The T-1000's backup plan was to capture Sarah, to use her as bait in his effort to catch John. John doesn't take the bait; he recognizes that his one hope is to survive. His mother, perhaps aided by police or military authorities, manages to destroy the T-1000 before it reaches John. However, she dies in the process. Thus at the end of this altered timeline, John determines once again to save his mother. Only the stakes are higher. If in sending help for his mother he opens the door for the Terminator to kill him, he has lost everything, even time itself. So he sends the original model Terminator back to protect himself, knowing that as a boy he would have saved her if he could have, and knowing that if he succeeded he would know that he saved his mother because he sent the Terminator back, and so would do so at the right time. The future is preserved.

Back to top of page.


Fourth Change: Rebels
Unfortunately, Sarah doesn't realize the complications of temporal anomalies. She thoroughly questions the machine, discovering how Skynet comes to be. No one realizes that all of this information is based on a time line which has been altered not less than five times from the original. All of the critical facts have changed. Relying on this information, Sarah Conner destroys all trace of the Skynet project at Cyberdyne, and (with a lot of help) both Terminator machines. She believes that she has stopped the war.

She has not stopped the war. She has merely created a new timeline. The company which originally invented Skynet will now invent it on the original schedule. The destruction of Cyberdyne and the original terminator will only shift the construction of Skynet back to its original timetable, unknown to us but undoubtedly slightly later.

She may have also changed her information. In this timeline she by destroying Cyberdyne has thrown Skynet back to the unknown company, but the sequence is again fforced to repeat itself. Just as in this timeline the Terminator has no knowledge of the company who would have developed Skynet had Cyberdyne failed, so too the Terminator who returns at the end of this link would know only that Sarah Conner destroyed the wrong company: it was not Cyberdyne but its competitor who created Skynet.

Here our story halts. We are in the second leg of our fourth temporal anomaly. To review, the first terminator killed Sarah after John was born, creating an N-Jump. Reese saved Sarah, but jump-started Skynet, creating a second anomaly, but because at the end of the second leg of this anomaly John sends Reese back not to save his mother but to become his father, and information flows in a circuitous manner from the future to the past to the future, this becomes a sawtooth snap resolved into an N-Jump escape. The T-1000 returns and, unable to kill John, instead kills his mother, creating a third anomaly, also an N-Jump. The fourth anomaly begins when the original terminator returns, ostensibly to protect John but actually to save Sarah; we are in the second leg of this anomaly. But this anomaly is going to become a problem, not because a person or a machine returned to the present, but because information did.


The problem begins when Sarah Conner questions the Terminator about the future. Changes begin to occur when Sarah Conner begins to question the Terminator regarding the future, because she in the segment we saw has changed that future, and so the machine's information regarding its past should be different. According to its version of the future, Sarah Conner attacked and destroyed Cyberdyne Systems, for no apparent reason. John Conner knows this; he knows it was based on information from the terminator which for some reason proved incorrect. Another company, the company which built the terminator, was responsible for the development of Skynet, which went on line at a date later than Sarah expected. Sarah would probably, and foolishly, attempt to destroy the research at that other facility as well. This would be more difficult, since that is genuine original research, not based on the study of pieces of a machine from the future; therefore, it will be spread out, stored in pieces in dozens of separate databases throughout the company, duplicated in the files of several individuals in several departments. However, she is obsessed with stopping the war, so she will probably try.

If she succeeds, an infinity loop is created--an extremely complex one. She is not only still in the C-D segment of the fourth anomaly, she is still in the C-D segment of every anomaly, including the first. If she stops the war, time is thrown back to the first segment, the segment in which she knew nothing and made no effort to stop anything, the segment in which she had a child of unknown name by a man not mentioned in the stories, and an unknown company launched a system called Skynet which became sentient and started a war to protect itself. All of time is caught in the loop. Let us hope she cannot stop the war.


It doesn't matter whether in this segment she destroys Cyberdyne. If she destroys the other company, the original terminator cannot come back, therefore Cyberdyne cannot have the pieces of the technology from the future, therefore neither company can create Skynet, war is averted, and time is restored to the original segment in which war is inevitable: the loop occurs.

If she fails, the original company continues on the original schedule. Cyberdyne is out of the picture, immaterial to the future. The war comes on the never-revealed original date, and the future is preserved. All that matters is that the war must begin, and Sarah Conner's child must survive to become SkyNet's enemy.
...
This Temporal Anomalies web site was launched years ago on another server with a page discussing Terminator and its sequel, Terminator 2: Judgment Day. Finally we come to the third, and presumably final, installment in the series. There is much to like about this film, although it is not without its problems.

Personal Asides
This is the first time a sequel has been made to a time travel movie about which we had already written an analysis, and it happens that in our earlier analysis we also made a few suggestions concerning what a sequel would have to do. Several people have written to say that it appears they took my advice. I am flattered. I have no proof of that, and no one asked my opinion officially. I am aware that my analysis was being discussed on an official forum, but only, as far as I know, by fans, and probably well after a script had been written. In any event, although some of what I suggested would have to be done was included in the film, the writers took it in a surprising direction. I am pleased with what they created, in terms of the overall story, although had I been consulting on the script I would have pointed out a few of the problems which are addressed here. It is, overall, a good time travel story, which errs in minor ways.

I also feel I ought to apologize for the delay in bringing this page to the site. It was probably over a year ago that someone sent me a copy of this film, and I watched it, analyzed it, and wrote a complete analysis. This was followed almost immediately by a computer crash, destroying the file before it was uploaded or backed up (I am using more effective backup methods now) and leaving me with far more urgent problems than a lost analysis of a time travel movie. Since then one interruption after another has prevented me from making meaningful progress on replacing it. I hope this analysis will be as good as that.

Finally, I should acknowledge the contribution made to this analysis by John "A1nut" Cross, who wrote the analysis of Final Countdown on this site, and who has long been a fan of the theories and the applications. I cannot say whether any of his ideas made it into this page, but I can say that his editorial review avoided at least a few silly mistakes, and his persistent nagging for me to complete an analysis and prove that the events of the movie are possible was a major impetus in forcing me to look deeper into what is happening here, resulting in a solution that manages to resolve all the major issues.

It is a rather convoluted solution, in some ways, but hopefully it will be clear enough to make its point.

Back to top of page.


Our Story So Far
A great deal has happened in our two previous full length movies; however, rather than insist the reader refer to the previous analysis, it will be easier to provide a quick synopsis of what is known to this point. On the other hand, this synopsis will assume that the reader has some understanding of the theory of time used for these analyses. If you have not read at least a few of the other pages on this site, this part is likely to confuse you.

It was established that there must have been an original history in which no one traveled from the future to the past. That is inherent in the theory of time, that history must reach the point at which a traveler departs before, in a sequential sense, that traveler can arrive in the past. A time traveler cannot help but change history, because he cannot have been part of that history at all until the history has been written once. His presence causes it to be rewritten at least once.

In this original history, Sarah Connor must have had a child. It was not Kyle Reese's child; it might not even have been a boy. All that is certain is that a child was born. It is also certain that someone created something called Skynet, a computer system able to control all of our computerized weapons systems, and that at some point this system became self-aware, and turned against humanity. An unfathomable number of humans died, but Sarah Connor's child survived, and led a resistance of some sort which fought back and prevented Skynet's victory. At some point, however, Skynet decided that it had one good option, to send a machine to the past to destroy Sarah Connor's child. For whatever reason, it determined to do this by destroying Sarah Connor before the child was born. Thus a Terminator was sent back, and history changed completely.

When I say that in this timeline Kyle Reese did not come from the future to protect Sarah Connor, the immediate thought will be that Sarah will die. However, in response to that it is probable that the original terminator was not the T-800 of the first movie, but a much less developed model. The reasons for that are explained in the previous page. Somehow, Sarah Connor managed to escape and give birth to her child. A terminator is relentless, though, and eventually it would have caught up with Sarah Connor and killed her. This must have happened after the child was born. Otherwise, there would have been no child, and Skynet would have had no reason to send the T-800 back, and we would fall into an infinity loop. Because Sarah was killed by the Terminator, Sarah's child would have sent Kyle Reese back to protect her; there is no good reason to have sent someone back to protect Sarah Connor if Sarah Connor survived (something already known from that moment in the future which immediately follows the departure of the Terminator), so Sarah must have been killed. This arrival of Kyle Reese in the past is the second time history changes completely, the third version of time.

The situation is severely muddied by the return of Kyle Reese, and this could have derailed history completely. Kyle tells Sarah much about the future; he also displaces the original father of Sarah's child, fathering the John Connor familiar to us. This sets up an escalating cycle, because Sarah will prepare John in ways she would not have done in previous histories. At the same time, working together Kyle and Sarah destroy the T-800 and leave its parts inside the factory of Cyberdyne Systems. This means that Cyberdyne is now examining the parts of the destroyed machine, and developing computer hardware unknown in previous histories.

It is at this point that Skynet becomes a product of Cyberdyne Systems. All history is rewritten to include this, because the Terminator that is sent back to kill Sarah is the more advanced model that benefits from the work done by Cyberdyne. As explained previously, Cyberdyne displaces the original creator of Skynet, getting it online sooner and with greater power.

It was into that history that Skynet sent the T-1000, this time to kill John Connor as a boy. Again, as explained in the other page, it fails to kill John, but it captures and kills Sarah, and so John feels the necessity to risk changing history by sending back the T-800 ostensibly to protect himself but actually to save his mother. This succeeds. Sarah Connor then pumps the T-800 for information, and destroys the Cyberdyne materials. This, we said, means that Cyberdyne does not create Skynet, but that whoever created it originally now does so again, on the original but long-lost timetable.

There is an extra wrinkle in this, concerning what John Connor had to do so that his mother would not cause an infinity loop. Sarah Connor has altered the future based on knowledge from the future, and so undone the knowledge on which she acted. We would expect that the information available to the T-800 now would concern the other developer, the one who originally and now again creates Skynet. John Connor can avoid this only by programming the T-800 with what is now false information, the history of the world that he was given by the T-800 at that time, which he now knows is not what actually happened. He must have recognized this necessity and done this, to avoid the infinity loop.

In essence, this is where the story paused; it is from all this that the story continues.

Back to top of page.


The Original Inventor
What this film gives us is the identity of the original inventor; it also gives us an entirely different kind of Skynet system. The first point was exactly what we needed; the second is interesting and unexpected, but quite workable.

In our original analysis, we faced the question of what would have happened had no one traveled from the future. That is not this scenario. John Connor has been born, son of Kyle Reese. The T-800 has come back twice and the T-1000 has also made its appearance. However, events have happened in this timeline which would undo the early development of Skynet and return it to the original history, in which someone other than Cyberdyne creates and launches Skynet at a later date. We now find that this someone is the Autonomous Weapons Division of the Cyber Research Systems branch of the United States Air Force, under the direction of General Robert Brewster.

The other difference here, though, is that Skynet is no longer a hardware system. There is certainly a great deal of hardware under the control of Skynet, but it is the software that is the essence of the system. Of course, Skynet was always inherently a mix of hardware and software, but now the software dominates, the hardware being the network of computers and Internet servers around the world now at the command of a system that is part virus and part artificially intelligent antivirus.

That matters within the story, because it is impossible to destroy Skynet once it has launched. John Connor does not understand that until the very end. However, that aspect of the problem saves the story. Had it been possible to stop the disaster ahead, John Connor would have stopped it. Had he done so, time would have been caught in an infinity loop, the future being undone based on knowledge from the future, the knowledge being thus undone restoring the future to what it was.

Back to top of page.


After Judgment Day
The third movie gives us two more anomalies, that is, events in which someone or something travels from the future to the past and so changes history, forcing it to be rewritten. However, what we have not yet done is establish exactly what must have happened in what we must loosely call the original timeline of this film, that is, that history which flowed from the destruction of the T-1000 in the previous movie. We were able to extrapolate a great deal of this in our previous analysis, but this movie gives us some essential detail.

We are immediately told that John Connor vanished "from the grid", that is, he reordered his life such that no one would be able to find him. There were no school records, no utility company listings, no property deeds, no registrations of his identity anywhere. (This suggests that the motorcycle he is riding at the beginning is not his, or at least that it is not registered in the United States, and that he does not have a legal drivers license; however, it is not impossible for someone to hold false license and registration, a fact attested by the number of hispanics in our southern New Jersey community who have Pennsylvania license plates on their vehicles.) This is not terribly important in this particular timeline, because although Skynet is seeking him, it will have to create a new history to target him.

There is no reason to think that John Connor would not have wiped out on the road, or that he would have done anything other than break into that veterinary clinic to acquire medicine and supplies to tend to his wounds. Betsy's cat will still get sick, and Katherine "Kate" Brewster will respond, finding the stranger in the clinic and locking him in the kennel. At that point, she manages to treat Betsy's cat and send the woman home, and then returns to talk with the familiar stranger whom she has by now correctly identified as the John Connor she once knew, who has some very strange stories to tell.

We know that these two get together. We can only speculate how. Perhaps when he tells her that he didn't kill his foster parents, she offers to take him out for some breakfast, and they start talking. He unfolds bits of his story. Probably he does not tell her that the people after him are machines from the future, but he does tell her that they tried to kill him the day after he was with her in Mike Kripke's basement, and that this other guy saved him and his mother, who had been locked up because these same people had tried to kill her but no one believed her.

The most difficult parts of this scenario are not that rough, really. We wonder how Kate and John survive, and why Kate never marries Scott. However, everything that happens to them happens in a single day. By the time they have finished breakfast, already the cell phones are out, the credit card machines are down--the virus that will fuse with the program to create the monster is doing its damage. John Connor is paranoid. He comes by this honestly. He knows this is imposing, but could Kate possibly drive him far, far away from this place--like to Mexico, or somewhere really rural.

While they are there, General Brewster launches Skynet to stop the virus, and the results are disastrous. Before Kate can leave John and drive back to Los Angeles, the missiles have launched, the disaster he has feared has come upon them, and Scott is an ashen memory.

Of course, there are people who somehow remarkably survive the blast. It is confusing how Jose Barrera and Elizabeth and William Anderson manage to live through the blast that destroys their city. However, they did. When John and Kate return, they will find these people alive.

I should note that this scenario falls apart if we believe that Skynet launched the virus that leads to its own creation. There is, however, no evidence that this is so. Although no one ever determines the origin of that virus, it did not come from the future.

It is also significant that in this play of the scenario, the autonomous weapons that play such a colorful role in the movie remain dormant for quite some time. There is no particular need to launch them. When General Brewster loses control of Skynet, he attempts to regain it, but there is not much urgency to this. He has no reason to believe that it is going to launch the missiles that destroy the world. It is at that point a serious problem, but not an insurmountable one.

All of this leads to that bleak future, the sending of several other time travelers, and a rather complex set of updated timelines. Then at some point in the future, a T-800 kills John Connor and is captured. At about that same time, Skynet makes another run at the past, this time sending back its most advanced machine, the T-X, described at one point as a terminator that terminates other terminators.

Back to top of page.


Skynet's Action
Why does Skynet send the T-X back, if John is dead?

There are actually two questions in this, one an obvious question with a deeply obscured and convoluted answer, the other a completely obscure question with a very simple answer.

The simple question is, if John Connor is now dead, why bother trying to kill him by sending someone back to the past? The problem is solved, right? What benefit is there to destroying the man who is now dead? The basic answer must be that John is no longer relevant when he is killed. Like all good leaders, he has made himself superfluous, redundant, unnecessary. The rebellion no longer needs him, and can function just as effectively without him. The opportunity to benefit from killing John Connor has long passed, and this one man is not important to the war effort any more.

This, though, opens the question, why bother to kill him if he is irrelevant? Certainly it is a moral victory. It will be a blow to the morale of the rebels for their revered iconic leader to have been assassinated so, even if it makes no difference to their organization. Yet there is another answer to this. Skynet is learning from the past. Every time it sends something back to kill John Connor, John Connor sends something back to protect himself. Thus Skynet has decided to try once more to kill John Connor before he matters, but this time has succeeded in killing him in the future before he can respond to this attempt on his life.

Of course, Kate Connor sends the T-800 back, so the tactic did not quite work as hoped. Still, Skynet does not actually know but only guesses who is sending help to the past, and since it cannot know at the moment that it kills John that it is Kate who will send the terminator, it cannot adjust its plan for that outcome.

Why, though, does it send the T-X, specifically? The answer to that is, it doesn't. It has been fighting a war in which resources are critical. There is no reason to send something as powerful as the T-X back. Note that the T-800 identifies the T-X as an anti-terminator terminator, saying that its own presence in the past has been anticipated. It probably was not anticipated. Skynet thought that the assassination of John Connor would prevent the rebellion from sending help to the past. It sent the least potent machine it believed would be effective in locating and terminating the members of John's team, which is probably another T-800. It was not until history played out a couple of times and the presence of Kate's T-800 in the past became evident that Skynet upgraded to the T-X, in a sense "retroactively anticipating" the presence of the other machine and reacting to it.

Thus we have the first change to history, as a terminator arrives in the past, sent by Skynet to terminate members of John Connor's team, including his wife Katherine Brewster, before they can become members of his team.

Back to top of page.


Altered and Erased
This is by far the most difficult of timelines to reconstruct, because it is the most improbable and yet the most necessary. It must be that a terminator is sent back to kill John Connor's lieutenants, including Katherine Brewster, and that it fails to do this, despite the fact that John Connor walks right into the middle of this and no help arrives.

Even for veteran readers of this site, that may require a bit of explanation.

We know that the time travel departures occur in sequence. That means before the T-800 can be sent back to protect John and Kate, the T-X must have been sent back to destroy them. However, once the T-X has been sent to the past, whatever it is "going to do" it has, from the perspective of the future, already done. If it has failed in its mission, then a moment or a day or a year later (it's all the same) there would be no particular reason to send anyone back to thwart it. However, if it succeeds, then the people it has killed are no longer among those who survived the disaster, and no longer on its roster of people it must kill. Further, if it kills Katherine Brewster and John Connor, then neither of them will be alive to send the T-800 back to protect themselves. This is a separate problem that will have to be addressed before moving to the next timeline, but for the moment it means that the T-X, as incredible as this seems, must have failed to kill Kate and John.

This supports our contention that T-X was not sent. Remember, when John Connor describes the T-X as a terminator that terminates terminators, the context includes the statement that the presence of the T-800 in this timeline had been anticipated. Perhaps it had been recognized retrospectively.


This requires a bit of temporal wrangling, but it is a perfectly reasonable scenario. We suppose that at a specific moment in time, Skynet sends back a terminator to track down and destroy John's lieutenants. It does not expect these to be difficult, and so does not waste resources, sending perhaps a T-800. The T-800 is not so bright. It tries to get Kate but hits the wrong target, and since it can't do that cute DNA identification by taste trick, it takes a bit longer to determine that Betsy was not Kate. A T-800 is also quite recognizable to John, and with one person shot, one recognizable terminator in the area, and one maiden in distress, he's going to take to the road. The problems are not so bad as they are in the movie, because the T-800 has few of the tricks used by the T-X in her pursuit. John and Kate might escape with their lives and flee to Mexico. The T-800 is still looking for them in Los Angeles, and gets destroyed in the blast. The rest of the scenario is very like the previous history.

Why, though, does Kate send the T-800 back to protect herself and John, all that far in the future? She must already know that they are not going to be killed by the terminator, because they are alive after that terminator leaves the future.


The answer this time lies in the list of targets the T-800 mentions when telling them of the lieutenants the T-X would be killing. One of these is General Robert Brewster. Even apart from the attack on the Cyber Research Systems facility, it is unlikely in the extreme that he would have survived the first wave of missiles. He is in a critical military facility, and will not have sufficient notice to reach Crystal Peak even if he abandons his efforts to regain control of Skynet. General Robert Brewster is not one of John Connor's lieutenants after the initial battle. Why, then, is he a target?

Here, the evident answer is that Skynet recognizes that sending back anything might create problems. It needs Brewster to live long enough to give it life, but not long enough to figure out how to kill it. It has put the general on the target list for this reason. John and Kate manage to escape, but whatever machine this was, it succeeded in killing Kate's father.

Then, just as she saw John send the T-800 back to protect his younger self so that his mother would be saved, she in turn sends back the T-800 to protect herself and her husband so that her father would be saved. This fails, but it has another effect.

Back to top of page.


Terminator 800
Perhaps a few hours after Skynet sends its machine to the past, Kate Brewster Connor sends her captured and reprogrammed T-800 back to protect her and John, and in the process to protect her father. It succeeds. It is possible even that it destroys the other terminator before General Brewster is killed--but not before the General is threatened. However, General Brewster does not succeed in shutting down Skynet. Perhaps he tells his daughter to take John (whom I think he must mistake for Scott, the fiance he has not met) and flee to the mountain, assuring them that he will follow. He never makes it, but is killed in that first blast.

However, as time advances we eventually reach the moment when Skynet sends back its terminator to kill those lieutenants. It now knows that a T-800 will be sent back, because we are in the history in which a T-800 was sent back. It thus replaces its terminator with the T-X, a model specifically designed to destroy other terminators, and thus the logical choice only if it is anticipating the presence of the T-800 in the past. The T-X is a much more formiddable killing machine, and John and Kate could not reasonably escape it on their own--but they do not have to do so. Because the T-X is sent back in the timeline created by sending back the T-800 after it, it arrives in a place in history that can only lead to the arrival of the T-800. Kate and John have help on schedule, and the events of the movie now unfold as shown.

It has been suggested that there is a problem with the T-800 knowing that Sarah Connor's coffin was filled with weapons. After all, John did not know this of his mother's coffin, and Kate certainly did not know, so who told the terminator? It is not so big a problem as suggested. Someone somewhere knows that there are weapons in that coffin; it is probably one of Sarah Connor's Mexican contacts. When John starts organizing the resistance, he will undoubtedly look for the people his mother knew, and particularly those whom he knew before. At that point he will learn about the weapons in the coffin. Since by that time he will be with Kate, she also will learn about it, including the place where these were hidden. As long as they do not attempt to recover those weapons in the future, no problem is created.

Once the location of the weapons has been passed to Kate and programmed into the T-800, the original source is irrelevant. After all, at the moment the terminator reveals that there are weapons in that coffin, John and Kate both know exactly where those weapons are. No one has to tell Kate about the weapons for her to know to include that information in the T-800 program years in the future. She knows they were there.

Back to top of page.


Killed By Minutia
All is not well, however. As the watch became the death of the story in Somewhere in Time, so here it is the little things that become the big problems.

While in the past, the T-X kills seven specific people. Three of them don't matter. It is already clear that Scott did not survive the disaster in the original history. General Brewster could not survive in any history, and we have discussed him already. There are also cat owner Betsy, and the woman in the sports car, whose deaths are likely to be a few hours premature. However, three people are killed who cannot be killed.

Even here, two of these might not be a problem.

The first of our victims is Jose Barrera. The T-X has downloaded information concerning his probable location by contacting the school database. This is something that another terminator probably could not have done, so Jose probably was not killed in any previous timeline. In this timeline, however, she has him on her list of targets, finds him at the fast food restaurant where he works, and as she passes through the drive-up window puts several bullets into him.

Although the chance of survival is slim, it is not none. Jose is not alone, and emergency services will be called. He will be rushed to intensive care, and could survive. Since he was standing in the window, it is probable that not all the bullets hit him, and that he fell to the floor. She did not confirm his death. Further, someone familiar with firearms tells me that the pistol in question appears to be a 9mm Baretta, and it is quite possible to survive several direct hits at close range with such a gun. Jose may have survived the shooting.

Of course, for him to have survived the bombing, he will have to have been moved out of the city the next day. This is highly unlikely, but again not impossible. If his family is Mexican, they may have decided that Los Angeles is too dangerous, and taken him back to Mexico once he was stable.

The problem, of course, is that if the T-X has killed Jose Barrera, then when the T-X is sent back Jose Barrera will not be on its hit list, because he will never have become one of John Connor's lieutenants. Thus he has to survive in order to be targeted, but if he is not targeted he will survive to be targeted. His death would create an infinity loop.

Elizabeth Anderson also presents this problem. It seem unlikely in the extreme that having determined from her brother that the girl is upstairs, the T-X would not have gone upstairs and killed her. On the other hand, gunshots downstairs are going to create a major panic, and Elizabeth might find a way out of the house. If she is upstairs with friends, the T-X might kill the friend believing it to be Elizabeth, not check the identity (the T-X might not have DNA samples on all of its targets), and leave.

The one that is most problematic, though, is William Anderson. Bill answers the door, and is shot several times at point blank range. The T-X then examines him before heading for the stairs. It does not seem possible that William Anderson survived, and if he did not survive, he could not be on the hit list, and if he is not on the hit list he was not shot, and if he was not shot he was on the hit list. We have an infinity loop based on the death of one boy.

It could be a case of mistaken identity. Anderson is not that uncommon a name, and William is extremely common. Just as the T-800 killed several Sarah Connors before coming after the right one, it might be that this is not the William Anderson who matters. The problem is that even though Elizabeth is also a fairly common name, it is very unlikely that there would be two brother-sister pairs named William and Elizabeth Anderson in the same location. Even if we include the possibility that the William and Elizabeth Anderson who should have been sought were husband and wife, not brother and sister, the odds are still very low. Still, if the T-X has actually killed one of its intended targets based on the knowledge that that person would live to become a danger to Skynet, the future is undone. It is a complicated loop, to be sure, because the T-800 has to leave in order for the T-X to have been sent, and we are going to be looping through several timelines in a rather unstable fashion, but we can no longer have a future beyond this point.

These, though, are the minor details. William and Elizabeth Anderson might be cases of mistaken identity. Some of John Connor's lieutenants might have taken false names, names of people they knew would die in the past, to prevent themselves from being targeted in precisely this way.

The same cannot be said of Jose Barrera. The T-X confirms his identity by means of a retinal scan. You cannot persuade me that a high school graduate working in the fast food industry has his retinal pattern on file somewhere. This retinal image must have been obtained after Jose became an important part of the resistance, specifically to identify him in the past. Skynet has matched Jose Barrera to his future self, making this a positive identification on this count. At the same time, that the T-X does not attempt a retinal scan on Betsy whom it thinks is Kate, nor on William Anderson, suggests that it does not have retinal images for all of those on its list.

It also complicates the Barrera situation because if he was identified by a retinal image taken in the future, his death in the past will mean he cannot be identified by that means. If he dies, it means an infinity loop for a long list of reasons. For the movie to work, he must survive.

There is at least a fair probability that being hospitalized would improve his chance of survival. Hospitals are generally well-built solid structures which under Geneva Convention guidelines must be constructed far from legitimate military targets. Not everyone in the blast area will die. The further removed you are from the blast center and the more cover you have against it, the greater your chance to survive. Thus the one victim most positively identified is also the one with the best chance of having survived, all factors considered.

I would rather conclude that John's lieutenants were not killed, because then I have a working time travel story and an excellent ending to a remarkable series. It has taken quite a bit of work to unravel it favorably, and it requires a few remarkable coincidences, but the question is not whether this is a probable story, but whether it is a possible one. The answer appears to be yes.

If Roland sees this: Good call, I didn't think of the spoilers in those. Sorry.

Dervag
2009-11-26, 03:18 AM
Ouch. You probably shouldn't bring that movie up in a time travel thread. It's just plain crazy. The first movie has a stable time loop, the second movie switches over to alternate timelines, the third movie contains bald-faced fatalism, and the fourth movie somehow manages to combine everything into a convoluted mess.

Yeah, fun mindless action (for the first two movies anyway), but not something you want to try and analyze.Sorry. It was a joke. Termination being the one thing that neither a stable time loop, nor the Terminator movies' take on time travel contains.

Also, the whole "robot war" scenario suggested it.

Seriously, though, what do you think of the idea, not the joke?

Serpentine
2009-11-26, 03:51 AM
Guys... Someone did an in-depth analysis of Terminator time-travel (although not the 4th movie). He found it plausible. It's right there for you to read :smalltongue:

Soras Teva Gee
2009-11-26, 04:59 AM
Guys... Someone did an in-depth analysis of Terminator time-travel (although not the 4th movie). He found it plausible. It's right there for you to read :smalltongue:

And its truly a case of tl:dr for me, mostly because the more I read the less I saw of any particular insights.

Sorry but I always find using proxies in place of one's own commentary at best uninteresting. Perhaps you could give us some highlights or key points on why Terminator's time travel is more plausible then any given particular case.

Serpentine
2009-11-26, 05:31 AM
Nah, I'm not clever enough for time travel. I just thought his stuff would interest the people who are clever enough to wrap their heads around it, and specifically with that one wanted to point out that, contrary to the claim that "You probably shouldn't bring that movie up in a time travel thread. It's just plain crazy... Yeah, fun mindless action (for the first two movies anyway), but not something you want to try and analyze.", someone has done so, and in a great deal of detail, and worked out many of its problems.

Nerd-o-rama
2009-11-26, 01:02 PM
I think of Time Travel in Popular Fiction not as a proxy for my own ideas, but as third-party research done by someone who can, at least, be considered an expert in this particular aspect of pop culture.

Also note that his analysis of Terminator 3, and indeed almost every movie he views as plausible, requires the cast and space-time itself to have jumped through a great deal of hoops not even remotely presented in the movie or even considered by the scriptwriters. It's more a matter of "can this plot be justified?" rather than "is this plot justified as presented in the film?"

Soras Teva Gee
2009-11-26, 01:58 PM
Bizarre as it sounds I think Futurama's Bender's Big Score is the only story I've seen where time travel was really thought out to make a sort of sense by conscious effort rather then happy accident.

Nerd-o-rama
2009-11-26, 03:57 PM
...did we watch the same movie? None of that made sense. Amusing, though.

I will give credit to the episode Roswell That Ends Well for establishing a stable time loop, at least in regards to the main paradoxical conflict.

Mewtarthio
2009-11-26, 04:22 PM
Also note that his analysis of Terminator 3, and indeed almost every movie he views as plausible, requires the cast and space-time itself to have jumped through a great deal of hoops not even remotely presented in the movie or even considered by the scriptwriters. It's more a matter of "can this plot be justified?" rather than "is this plot justified as presented in the film?"

Exactly. It's fun to try and make things work out, and Terminator does indeed make perfect sense if you assume that the John Connor who first sent Reese back in time is not the John Connor who Reese fathered (though how he'd have grown up prepared to fight Skynet if Sarah didn't know about it ahead of time is beyond me). The trouble is that the first movie presents itself as a stable time loop: You can assume the two Johns are different, but that's clearly not the movie's intent. Then the second movie switches over to alternate timelines with no explanation. They don't even bother retconning the first movie: They clearly just didn't care what time travel system they were using.

Soras Teva Gee
2009-11-27, 11:25 AM
Well you can always have a stable time loop established within any way of handling time travel since it by definition doesn't change the past and resolving alteration is how most systems differentiate. That said needing to preserve said loop I have no idea if Terminator's holds up.

Dervag
2009-11-28, 03:09 AM
Yeah. The trouble with stable loops is that they violate our intuition harder than most other forms of time travel, because there is no way to fit them into the linear-causation model. The question "but what started the loop" is the wrong question, because within the "reality" of the timeline there is not and never was an outside force that created the loop.

If John Connor sent back Some Guy to protect his mom, and (Time-Travelled) Some Guy ends up fathering John Connor-Prime, and John Connor-Prime later sends back Some Guy to protect his mom... there never was an original John Connor, only John Connor-Prime. Just as in my earlier argument, A' is an illusion and there is only A and A''.

From our point of view A' existed, but our point of view lies outside the universe, and we can see all sorts of things that aren't real in that universe's frame of reference.

Soras Teva Gee
2009-11-28, 03:42 AM
I think the counter-intuitive nature of stable time loops has (psychologically) less to do with linear causality and more to do with people still holding on to the notion that time is somehow absolute. Which continues despite Einstein throwing it out before most people breathing air today were born. (Though the two are not entirely unrelated...)

Course its not like I can wrap my brain painlessly around time travel without imagining it from an outside perspective in a reduced from that while I can visualize can't actually exist, but we've got no real reason to suppose it does. Does time happen in some sort of over-dimensional medium/space?

Nerd-o-rama
2009-11-28, 12:45 PM
In order to time travel in the first place, assuming our current model of a 4-17ish-or-maybe-infinite-dimensional universe holds, you'd have to be moving in some dimension other than the normal three we can perceive. It's not a "over-dimensional medium/space", it's just another axis on a four-dimensional Cartesian hypercube. Or for all I know, another three axes on a six-dimensional Cartesian 6-cube.

And yeah, calling 3+-dimensional fields Cartesian is a stretch, but you know what I mean.

Golden-Esque
2009-11-28, 01:26 PM
I abhor time travel plots. There are several "internally consistent" time travel types... when you get a time travel story it typically uses an inherently inconsistent model or it switches models midway.
Even if it manages to avoid the most obvious pot holes, it still ends up full of plot holes...

So here is the challenge. Can someone name any single story, comic, book, movie, or whatever; which contains time travel and is not left with gaping plot holes caused by said time travel in the first place?

When you post the name of such a story, expect it to be scrutinized by people hell bent on disproving your claim.

Also, feel free to suggest "solutions" to said plot holes that will not wreck the original plot.

You might enjoy the novel "When You Reach Me". It's one of the most consistent examples of Time Travel I've ever seen.

PirateMonk
2009-11-28, 05:24 PM
This still fails for a single-universe model. In multiple timelines, there is no loop, just a spiral that looks like a loop if you try to superimpose all the myriad timelines. But in a single-universe setting, we are left with the contradiction that my grandfather receives a letter with one mark and a letter with two marks and a letter with three marks and... and so on... but still only receives one letter.

That applies to any attempt to change history in a single-universe environment. For example, Lincoln is both shot and not shot. So there are three possibilities: any attempt to give the letter to your grandfather and get it back with one more mark fails because it would change the past, a new timeline and infinite spiral are formed, or the act of taking it back in time, despite the fact that it is only taken back once, changes history in a way that changes history further, causing an unstable time loop.


The Many Worlds Interpretation (http://en.wikipedia.org/wiki/Many-worlds_interpretation) insists there are infinite alternate timelines in existence even without Time Travel. If one subscribes to it (as much of science does, since the only real alternative is a deterministic universe)

I was under the impression that the alternative is that certain events have certain probabilities of happening, and things which don't happen don't happen.

Volkov
2009-11-28, 05:27 PM
The Red Alert series can do it (almost anyway), but only because time travel is done once a game.

Soras Teva Gee
2009-11-28, 05:56 PM
In order to time travel in the first place, assuming our current model of a 4-17ish-or-maybe-infinite-dimensional universe holds, you'd have to be moving in some dimension other than the normal three we can perceive. It's not a "over-dimensional medium/space", it's just another axis on a four-dimensional Cartesian hypercube. Or for all I know, another three axes on a six-dimensional Cartesian 6-cube.

And yeah, calling 3+-dimensional fields Cartesian is a stretch, but you know what I mean.

Yeah that's what I was referring to. I'm way too over-skeptical on what science claims to say in this area though. A little past relativity and the basics of quantum mechanics everything become more a mathematical model I don't have the grounding to follow. And science can't seem to agree on anything for too long.

For comparison the Big Crunch and oscillating universe model were pretty big when I first looked into all this stuff, then that whole Dark Energy think percolated through. So yeah.

Nerd-o-rama
2009-11-29, 12:26 AM
Dark Energy is a stopgap that I don't think anyone seriously believes - it's just a "please hold" while science comes up with a better reason for an expanding universe that contradicts their previous math.

This is why I don't dismiss time travel out of hand - our knowledge of physics is being constantly revised, and it's in pretty rapid flux right now that we have two contradictory big, overall models of the universe thanks to quantum physics not agreeing with relativity and vice versa. It's pretty exciting and makes me wonder why I decided to go into business instead of academia.

GoC
2009-11-29, 05:35 AM
I abhor time travel plots. There are several "internally consistent" time travel types... when you get a time travel story it typically uses an inherently inconsistent model or it switches models midway.
Even if it manages to avoid the most obvious pot holes, it still ends up full of plot holes...
I have nothing to add except: QFT!

Volkov
2009-11-29, 09:24 AM
Dark Energy is a stopgap that I don't think anyone seriously believes - it's just a "please hold" while science comes up with a better reason for an expanding universe that contradicts their previous math.

This is why I don't dismiss time travel out of hand - our knowledge of physics is being constantly revised, and it's in pretty rapid flux right now that we have two contradictory big, overall models of the universe thanks to quantum physics not agreeing with relativity and vice versa. It's pretty exciting and makes me wonder why I decided to go into business instead of academia.

Dark energy is much more than a stop gap. It's widely supported by scientists, much like it's cousin, Dark Matter. After all, Dark energy accounts for %70 of the universe's mass and energy.

GoC
2009-11-29, 02:19 PM
Dark energy is much more than a stop gap. It's widely supported by scientists, much like it's cousin, Dark Matter. After all, Dark energy accounts for %70 of the universe's mass and energy.
It's a stopgap that's used by the majority of science. A way of saying "We don't know". Look past the name and at the substance and what do you see? Any predictions? No? Then it's a "We don't know" sign.

Devils_Advocate
2009-11-29, 03:21 PM
I honestly don't know. I tend to believe in a deterministic universe myself, but a lot of scientists hate the idea.
My understanding is that Many Worlds actually is a deterministic interpretation of Quantum Mechanics: It's the notion that every possible outcome of an event happens, instead of just one of the outcomes happening randomly.

Soras Teva Gee
2009-11-29, 03:56 PM
Dark Energy is a stopgap that I don't think anyone seriously believes - it's just a "please hold" while science comes up with a better reason for an expanding universe that contradicts their previous math.

This is why I don't dismiss time travel out of hand - our knowledge of physics is being constantly revised, and it's in pretty rapid flux right now that we have two contradictory big, overall models of the universe thanks to quantum physics not agreeing with relativity and vice versa. It's pretty exciting and makes me wonder why I decided to go into business instead of academia.

I don't dismiss time travel either. I just try to be only loosely associated with what we 'know' about the universe. And yes Dark Energy is a stopgap, but its one that says we've just looked below the waterline of the iceberg we're standing on. When science says the majority of stuff in the universe they can't see and have no real idea what it is... well I wouldn't be surprised to find that if by the time I die some current science goes the way of the ether.

But that's starting to get away from the topic.

Volkov
2009-11-29, 05:34 PM
It's a stopgap that's used by the majority of science. A way of saying "We don't know". Look past the name and at the substance and what do you see? Any predictions? No? Then it's a "We don't know" sign.

What is known is that it counteracts gravity to an incredible degree and is extremely common.

The Big Dice
2009-11-29, 06:21 PM
Dark Energy is a stopgap that I don't think anyone seriously believes - it's just a "please hold" while science comes up with a better reason for an expanding universe that contradicts their previous math.

I've got some serious issues with the whole Dark Energy thing. Particularly the 13.5 billion light year barrier. A lot like the Inflation aspect of the Big Bang theory, the observable evidence doesn't match the hypothesis. But rather than do like science teaches and search for a better hypothesis, ideas are brought in that are used to support the scientific status quo. Maybe the universe is simply a heck of a lot bigger than we are capable of perceiving.

There's two relevant quotes to the ongoing discussion, though. The first comes from Arthur C Clarke, who said "Not only is the universe stranger than we imagine, it is stranger than we can imagine." The second is from Stephen Hawking and goes something like: "If time travel is possible, where are the tourists from the future?"

All that said, I'm amazed that a thread about time travel stories has got to four pages and nobody mentioned Doctor Who. The episode Blink in particular is a great time travel story, showing some of the complexities that the life of a time traveller can bring. Especially when from one perspective A causes B which leads to C, but from another perspective the chain of cause and effect is in a different order.

Volkov
2009-11-29, 07:46 PM
I've got some serious issues with the whole Dark Energy thing. Particularly the 13.5 billion light year barrier. A lot like the Inflation aspect of the Big Bang theory, the observable evidence doesn't match the hypothesis. But rather than do like science teaches and search for a better hypothesis, ideas are brought in that are used to support the scientific status quo. Maybe the universe is simply a heck of a lot bigger than we are capable of perceiving.

There's two relevant quotes to the ongoing discussion, though. The first comes from Arthur C Clarke, who said "Not only is the universe stranger than we imagine, it is stranger than we can imagine." The second is from Stephen Hawking and goes something like: "If time travel is possible, where are the tourists from the future?"

All that said, I'm amazed that a thread about time travel stories has got to four pages and nobody mentioned Doctor Who. The episode Blink in particular is a great time travel story, showing some of the complexities that the life of a time traveller can bring. Especially when from one perspective A causes B which leads to C, but from another perspective the chain of cause and effect is in a different order.
The universe is much larger than the barrier, and the barrier itself actually extends past 13.7 billion light years due to a somewhat hard to understand effect. Of course as an Herpetologist, I'm not qualified to speak much about that.

Dervag
2009-11-29, 08:04 PM
That applies to any attempt to change history in a single-universe environment. For example, Lincoln is both shot and not shot.Huh? That's the opposite of my point; in a single timeline Lincoln must either be shot or not be shot, and cannot possibly do both.


So there are three possibilities: any attempt to give the letter to your grandfather and get it back with one more mark fails because it would change the past, a new timeline and infinite spiral are formed, or the act of taking it back in time, despite the fact that it is only taken back once, changes history in a way that changes history further, causing an unstable time loop.Precisely. Except that "unstable time loop" may well be a meaningless concept, since it cannot be explained in the frame of reference of any single timeline. Stable time loops can, although they are confusing in a single-timeline frame. And infinite spirals through multiple timelines make sense locally for any given timeframe (the letter appears in your universe at iteration N and leaves at iteration N+1, never to return).

But unstable time loops don't work that way, because they violate the either/or condition that must hold when we make statements about specific events in a single timeline.


It's a stopgap that's used by the majority of science. A way of saying "We don't know". Look past the name and at the substance and what do you see? Any predictions? No? Then it's a "We don't know" sign.True, but it is more than an epicycle added to make the model look pretty: it is a real phenomenon that is as yet unexplained within the existing model, except speculatively.


I've got some serious issues with the whole Dark Energy thing. Particularly the 13.5 billion light year barrier. A lot like the Inflation aspect of the Big Bang theory, the observable evidence doesn't match the hypothesis. But rather than do like science teaches and search for a better hypothesis, ideas are brought in that are used to support the scientific status quo. Maybe the universe is simply a heck of a lot bigger than we are capable of perceiving.Could you explain in more detail? My response to this sort of remarks depends heavily on how much the person I'm talking to knows about general relativity.

If you are already fully familiar with the existing theory, and the numerous experimental supports for it, then I have one set of things to say to this; if you are not, I have a different set of things to say.

GoC
2009-11-29, 08:47 PM
True, but it is more than an epicycle added to make the model look pretty: it is a real phenomenon that is as yet unexplained within the existing model, except speculatively.
I never said otherwise.
I was pointing out that it's nothing more than a fancy label for a set of observations that aren't accounted for in the current model. While it (probably) does predict that we will continue to see gravitic anomalies, it is satisfied by pretty much any possible observation regarding them* and so it doesn't really add to our knowledge.
Compare things like GR or QM that made specific and surprising predictions that were then confirmed (they could also have been disproved by those observations going the other way).

*I'm speculating here but everything I've heard points in this direction.

Dervag
2009-11-30, 11:27 AM
I never said otherwise.
I was pointing out that it's nothing more than a fancy label for a set of observations that aren't accounted for in the current model. While it (probably) does predict that we will continue to see gravitic anomalies, it is satisfied by pretty much any possible observation regarding them* and so it doesn't really add to our knowledge.
Compare things like GR or QM that made specific and surprising predictions that were then confirmed (they could also have been disproved by those observations going the other way).

*I'm speculating here but everything I've heard points in this direction.The trick is that dark energy fits into the existing model; it is not a complete X-factor even if we can't explain where it comes from. I may be getting this wrong, but as I understand it the simplest way to explain dark energy is to posit that creating a new volume of space releases a certain amount of energy; thus the expansion process is to some extent self-powering.

Again, we can't explain where it comes from, but it's more than just a pure "we don't know what's going on here" label the way that, say, "cathode rays" were when they were first observed.

GoC
2009-11-30, 02:38 PM
Dervag: Ah thanks for enlightening me.:smallsmile:

The Big Dice
2009-11-30, 03:50 PM
If you are already fully familiar with the existing theory, and the numerous experimental supports for it, then I have one set of things to say to this; if you are not, I have a different set of things to say.

I've got a reasonably good layman's understanding of general and special relativity. I'm no physicist, so the maths is beyond me. But the concepts aren't completely beyond my understanding.

However, there are issues with the Big Bang model, especially the earliest times that are conjectured about. Particularly Inflation. And things like the Large Hadron Accelerator and other high energy experiments, while interesting in their own right, have been described by the people operating them as "Like trying to understand how a Swiss watch is made by firing one into another and then analysing the pieces."

Think on this, though. If the universe is 13-14.6 billion years old, as we get told by cosmologists, why is there a barrier at that distance at all?

Dervag
2009-12-01, 02:47 AM
I've got a reasonably good layman's understanding of general and special relativity. I'm no physicist, so the maths is beyond me. But the concepts aren't completely beyond my understanding.The danger created by this level of understanding is that the math is the proper language in which to describe the concepts. Without the math, you're stuck relying on translations, and if you get a bad translation*, you wind up with a misunderstanding of the underlying science as serious as what you get from watching a badly dubbed movie.

So it pays to be careful when making strong statements about the flaws in a scientific theory one cannot do the math for; I know I try to be careful about subjects like quantum field theory where my understanding does not include a grasp of the mathematics.

*(an inept science popularization, for instance)
_______


However, there are issues with the Big Bang model, especially the earliest times that are conjectured about. Particularly Inflation.In what way are there issues with inflation? Inflation explains a number of observed phenomena quite well; what issues does it raise that are of comparable weight to the ones it solves?


And things like the Large Hadron Accelerator and other high energy experiments, while interesting in their own right, have been described by the people operating them as "Like trying to understand how a Swiss watch is made by firing one into another and then analysing the pieces."This is a misleading statement. Accelerators have produced many experimentally testable results about the behavior of subatomic particles, and have been an extremely valuable tool for advancing our understanding of particle physics since the 1930s (earlier, if you count the "cathode ray" experiments that allowed us to discover the electron).

There's a big difference between unanswered questions and internal contradictions: our current model of the universe contains a number of questions, but very few (if any) contradictions. Hopefully, finding the answer to the questions will reveal contradictions that let us move to a higher level of physical understanding, but we're nowhere near the point of being able to say with confidence that this will happen.


Think on this, though. If the universe is 13-14.6 billion years old, as we get told by cosmologists, why is there a barrier at that distance at all?What barrier? There is no barrier; there's just a limit on how far we can see, because light from more distant bits of the universe hasn't had time to reach us yet. Wait a billion years and the "barrier" will be more remote, if you have the patience for it.

GoC
2009-12-01, 03:10 AM
What barrier? There is no barrier; there's just a limit on how far we can see, because light from more distant bits of the universe hasn't had time to reach us yet. Wait a billion years and the "barrier" will be more remote, if you have the patience for it.
I think he means the distance where nothing from that location could ever reach us now due to the universe expanding. Strangely enough I'm getting that distance at only 13 billion light-years in the present universe. Strange when considering the observable universe's 45 billion light year radius.:smallconfused:

EDIT: Yes, I know real astronomers use parsecs.:smalltongue:

Nerd-o-rama
2009-12-01, 11:43 AM
True, but it is more than an epicycle added to make the model look pretty: it is a real phenomenon that is as yet unexplained within the existing model, except speculatively.This is closer to what I meant - sorry for touching off this tangent and then ignoring it.

"Dark Energy" is an observable phenomenon, it's just quite badly explained, and points to a large gap in the understanding of our current scientific models; I believe, not unreasonably I think, that there's significantly more to the effect than "there's a large and measurable amount of something that counteracts gravity". The same is probably true of Dark Matter - at the very least, I'd expect such matter to have some measurable properties besides "we can't observe the matter itself, but it appears to have a lot of mass".

Hawriel
2009-12-01, 06:48 PM
Star Trek IV. Go back in time and get whales to make nice with the big alian space probe. No big holes. Just afew questions about circumstances that occured in the movie.

The klyngon phaser, comunicator, and possibly scanner (unless Uhura took that) was left behind on the USS Enterprise.

The transparint alluminum. Scotty gave it to a manufacturer. Bones pointed out the possable contamination of the time line. Scotty blew it off.

Even after the whales where brought back to the pressent, and talked to the alian probe. Two whales and a baby will die off quickly. They cannot create a lasting population. Unless a bunch of scientests dedicate their carears in making test tube whale babies, and smoothing out the problems with inbreading.

The dissapearince of the Oceanographer/biologist. Or what ever her job was. Then again thats just one more missing persons file in a very large city.

Dervag
2009-12-02, 12:23 PM
I think he means the distance where nothing from that location could ever reach us now due to the universe expanding. Strangely enough I'm getting that distance at only 13 billion light-years in the present universe. Strange when considering the observable universe's 45 billion light year radius.:smallconfused:The observable universe has such a large radius today, but the light that is reaching us now has only traveled 1.3*10^10 light years. The stuff that emitted it is much farther away now than it was when it did the emitting.

Effectively, what's happening is that the expansion of the universe constantly creates new space between any two points. If the two points are far enough apart, the amount of new space added every second is larger than the distance you can travel in one second, even at the speed of light... in which case that point is permanently out of your reach.


This is closer to what I meant - sorry for touching off this tangent and then ignoring it.

"Dark Energy" is an observable phenomenon, it's just quite badly explained, and points to a large gap in the understanding of our current scientific models; I believe, not unreasonably I think, that there's significantly more to the effect than "there's a large and measurable amount of something that counteracts gravity". The same is probably true of Dark Matter - at the very least, I'd expect such matter to have some measurable properties besides "we can't observe the matter itself, but it appears to have a lot of mass".The catch is that both dark energy and dark matter are extremely diffuse; if they were concentrated near us, we would know far more about them because we could observe their drastic effects on the more visible stuff around them.

It may very well turn out that dark matter and dark energy are only relevant on cosmic distance scales, although there is no way to prove this in advance except to note that no matter what we learn about them, it must still reduce to what we already know within the region we can easily describe. Dark matter might allow us to make Unobtanium, for instance, but if the normal concentrations of dark matter around Earth caused Unobtanium to appear in nature we would have noticed it long since.

Telonius
2009-12-02, 01:14 PM
Another potentially internally-consistent one: "Door Into Summer" by Heinlein. The time travel paradox is (possibly) resolved because of one item. Its existence depends on the main character not noticing it at a crucial point.

Talya
2009-12-02, 01:22 PM
I was under the impression that the alternative is that certain events have certain probabilities of happening, and things which don't happen don't happen.

Except in that case there are no real probabilities. There are only guarantees.

Let us make one and only one assumption - that there are natural laws of physics that remain consistent and all the universe(s) must follow. This is not guaranteed, it's just the only possibility wherein we can begin to logically discuss something of this nature. If the laws of physics change randomly or do not apply universally the same way, then we have no way to even begin to quantify things like this.

Let us imagine that some supercomputer of infinite power has a complete knowledge of both those natural laws, and the initial state of the universe as we know it at the point of The Big Bang (or whatever other start-of-creation scenario you happen to subscribe to.) With that information, that supercomputer would be able to predict the state of the universe down to the atomic level (and perhaps subatomic, depending on whether quantum particles are truly random or not) from conception until the eventual heat death of the universe. (and beyond, if it were relevant.) There would be no choices, no pivotal points upon which the direction of history or the universe could change, and free choice among sentients becomes nothing more than an illusion attributable to our limited understanding an capabilities of observation. What was, what is, and what will be are all just parts of an unimaginably complex equation.

Now, there is no such supercomputer, but the fact of the matter is there was a definite initial state for the universe, and if the universe does indeed follow consistant physical laws, then the end is already decided for us all. There can be no branching timelines, no alternate universes, no "possibilities," because everything is definite and determinate.

Introduce randomness into the equation, whether the metaphysical (but to my mind, preposterous) concept of "free choice," or simply quantum uncertainty with a physical random number generator to throw a wrench into things, and the idea of alternate reality gains traction. And in fact, that quantum uncertainty only really makes logical sense when alternate realities are accepted. How can the particle be in two mutually exclusive states (a superstate) at the same time, until measured? The point of measurement was the point where realities branched. Up until that point, with the particle in that superstate, the realities were identicle and could overlap, but at the point of relevance, when Schroedinger's box is opened, both states could no longer exist simultaneously, and reality was torn in two.

WalkingTarget
2009-12-02, 01:42 PM
I was a big fan of To Say Nothing of the Dog. I haven't read the other related things, but I liked the way they handled things.

Time travel was possible only if "significant" items/events aren't interfered with.

That is, you don't have to worry about the air in your lungs or the food you ate while in the past since a few oxygen/carbon/etc. molecules don't make much difference either way, but you can't normally bring macro-scale things back with you (of course, much like the "I, Robot" stories, the book deals with an apparent exception to the rules) and the reason there aren't thousands of time-tourists at significant historical events is that if you aim for them there is "slippage" and you wind up missing the target by enough time/distance that you can't interfere.

It's also absolutely hilarious.

Talya
2009-12-02, 01:46 PM
Time travel was possible only if "significant" items/events aren't interfered with.

That is, you don't have to worry about the air in your lungs or the food you ate while in the past since a few oxygen/carbon/etc. molecules don't make much difference either way...

I'm a strong believer in "The Butterfly Effect." There is no such thing as insignificant items or events.

WalkingTarget
2009-12-02, 01:56 PM
Let's just say that by the end of the book they've amended their understanding of the "rules" rather significantly. I'm trying to avoid spoilers here.

Dervag
2009-12-02, 04:44 PM
I'm a strong believer in "The Butterfly Effect." There is no such thing as insignificant items or events.It's questionable how large a butterfly needs to be in order to cause large changes instead of just getting damped out. Sufficiently small changes will be damped out, simply because their effects are suppressed by noise, or because there are forces at work that are fundamentally larger than the subtle change.

A butterfly flapping its wings in Manila might cause a hurricane in Cuba in four months' time, but it cannot prevent winter from changing into spring.

Talya
2009-12-02, 04:58 PM
A butterfly flapping its wings in Manila might cause a hurricane in Cuba in four months' time, but it cannot prevent winter from changing into spring.

And that hurricane might kill a man who would otherwise have gone on to be the great, great, great grandfather of Kang the Mighty, conqueror of the Galaxy.


Or something.

(This thread was getting too serious.)

Nerd-o-rama
2009-12-02, 05:07 PM
But it's also possible that the same sociopolitical and scientific situations could cause a leader like Kang the Mighty, conqueror of the Galaxy to arise at around the same time and do approximately the same things, even if he's named Kodos and was born a continent away to different parents. History would be changed (relative to the butterfly not flapping its wings), but not so you'd notice.

Douglas
2009-12-02, 05:26 PM
Ah, but it's also possible that Kang the Mighty would have been a strategic genius the likes of which only comes along once every several centuries, and his genius as a military commander was absolutely necessary for his conquest to succeed.

Nerd-o-rama
2009-12-02, 11:29 PM
Maybe Kodos had as much, or perhaps more, native intelligence, and went to the same military academy.

Really, we're going off into tons of what-ifs, though. I like to imagine history as a stream (not that I think time is anything that simple, I'm talking about human history as we see it). As the stream flows through it's course, it wears a path that's predicated on its environment and its own nature. Maybe you could change the course by dislodging a well-chosen pebble, but usually the pebbles just going to bounce along the bottom while the stream's course is affected by bigger things like ground faults, bigass rocks, and the local rainfall. If you want to change it, you're going to have to dig a big damn trench, or trench a big dam.

GoC
2009-12-03, 07:56 AM
The observable universe has such a large radius today, but the light that is reaching us now has only traveled 1.3*10^10 light years. The stuff that emitted it is much farther away now than it was when it did the emitting.
I know this. I am asking if 13 billion ly really is our "event horizon".

Dervag
2009-12-04, 01:32 PM
And that hurricane might kill a man who would otherwise have gone on to be the great, great, great grandfather of Kang the Mighty, conqueror of the Galaxy.

Or something.

(This thread was getting too serious.)Absolutely. The question is whether there is a thermal noise effect in history when it is viewed at long timescales. And the proposition that there is one is a completely untestable hypothesis except in a multiple-timeline time travel scenario.

Chaos theory does not guarantee that all small changes in initial conditions will produce disproportionate changes in final conditions; there can still be changes that make no difference or changes so small that they wind up in a linear regime (change in outcome is proportionate to change in initial conditions) instead of a chaotic nonlinear one (change in outcome is disproportionate).

======


I know this. I am asking if 13 billion ly really is our "event horizon".In the sense that it represents a range limit, such that objects beyond that distance from us today are permanently beyond our reach? I don't know. Let me do an extremely crude estimate.

OK, according to Dr. Wikipedia, the Hubble Constant, H, is roughly 75 km/s per megaparsec. Our "event horizon," the radius of the region accessible to us, is defined by the distance beyond which objects appear to be receding at the speed of light, c, which is 300000 km/s.

If the expansion of the universe were a constant, then the "event horizon" has a radius of:

r = c/H = (300000/75) megaparsecs
r = 4000 megaparsecs
r = (4*3.26) billion lightyears
r ~= 13 billion lightyears.

So yes, 13 billion lightyears is about right, give or take a little, assuming the expansion of the universe is occuring at a constant rate. It's not necessarily true that the rate is exactly constant, but it's close, so... yes. 13 billion light years.

This does not mean that we cannot see objects that are now more than 13 Gly* away as they were in the past, back when they were inside our event horizon. But things that happen more than 13 Gly away from us today will be forever invisible to us unless we contrive some method of travelling faster than light. Light spreading outwards from those events will find that after each second of travelling toward us, we are farther away than we were a second ago. Thus, it cannot catch up.

*gigalightyears
** This is how scientists actually answer the questions people ask them in contexts like this. A combination of improvisation, rounding, and reference to surprisingly easy-access sources.
________

Please note that the "galaxies receding" thing is a sort of optical illusion; what's really happening is that more space is being added in between the galaxies over time. Things end up farther apart than they used to be even though (and this is important) they are not being pushed by any physical force.